SUSIEG 2018 C Flashcards

1
Q

After an uncomplicated acute myocardial infarction (MI), under what circumstances should a patient be administered a graded exercise test (GXT) before hospital discharge?
A. Symptom-limited GXT at 10 days post MI.
B. Low level GXT at 4 to 6 days post MI.
C. GXT to 85% age predicted maximum HR 3 to 5 days post MI.
D. GXT to 75% age predicted maximum HR 4 to 6 days post MI.

A

Answer: B
Submaximal GXT can be administered before hospital discharge at 4 to 6 days post acute MI.
Low-level exercise testing provides data for recommendations for ADL and early ambulatory exercise therapy. This amount of activity doesn’t place too much demand on the healing myocardium (ACSM guidelines for Exercise Testing and Prescription, 8’ ed.).

How well did you know this?
1
Not at all
2
3
4
5
Perfectly
2
Q

A newborn is examined at birth using the APGAR test. Which of the following APGAR results is a likely indicator of potential neurological complications?
A. 3 at 10 minutes
B. 9 at 1 minute.
C. 8 at 1 minute.
D. 8 at 5 minutes.

A

Answer: A
The APGAR score is based on heart rate (HR), respiration, muscle tone, reflex irritability (grimace), and color (appearance). APGAR scores are routinely assigned at 1 and 5 minutes and occasionally at 10 minutes postbirth.

How well did you know this?
1
Not at all
2
3
4
5
Perfectly
3
Q

APT requested that a physical therapy assistant (PTA) perform ultrasound (US) to the shoulder of a patient. During the treatment session, the patient experienced an electrical shock. In which situation would the PT be responsible for any injury the patient might receive?
A. Faulty circuitry.
B. The PTA failing to use a ground fault interrupter (GFI).
C. The patient touching the US device during treatment.
D. The PT having instructed the PTA to use a device that had malfunctioned on the previous day.

A

Answer: D
The PT in this case correctly delegated the US treatment to the PTA. Every individual (PT, PTA) is liable for their own negligence; however, supervisors may assume liability of workers if they provide faulty supervision or inappropriate delegation of responsibilities (not evident in this case). PTs are liable for use of defective equipment if they contributed to its malfunction or continued to have it used in treatment without having it checked.

How well did you know this?
1
Not at all
2
3
4
5
Perfectly
4
Q

A patient with coronary artery disease received inpatient cardiac rehabilitation after a mild myocardial infarction (MI). The patient is now enrolled in an outpatient exercise class that utilizes intermittent training. What is the BEST initial spacing of exercise/rest intervals to safely stress the aerobic system?
A. 5:1
B. 1:1
C. 10:1
D. 2:1

A

Answer: D
Presuming that the exercise goals for inpatient cardiac rehabilitation are met, an exercise/rest ration of 2:1 can be used with this patient to begin exercise in an outpatient setting in a safe
manner.

How well did you know this?
1
Not at all
2
3
4
5
Perfectly
5
Q

What is the BEST initial intervention to improve functional mobility in an individual with a (stable humeral neck fracture?
A. Isometrics for all shoulder musculature.
B. Heat modalities
C. Active resistive range of motion (ROM)
D. Pendulum exercises.

A

Answer: D
This individual will typically be immobilized with a sling for a period of 6 weeks. After 1 week, the sling should be removed to have the patient perform pendulum exercises to prevent shoulder stiffness.

How well did you know this?
1
Not at all
2
3
4
5
Perfectly
6
Q

A patient with unilateral spondylolysis at L4 is preferred for physical therapy. The patient complains of generalized lower back pain when standing longer than 1 hour. Which (strengthening exercise is BEST fot the subacute phase of this patient’ rehabilitation?
A. Multifidi working from neutral to full extension
B. Abdominals working from neutral to full flexion.
C. Multifidi working from full flexion back to neutral.
D. Abdominals working from full extension to full flexion.

A

Answer: C
Performing strengthening exercise to the multifidi from flexion to neutral will not stress the pars defect.

How well did you know this?
1
Not at all
2
3
4
5
Perfectly
7
Q

A patient has fixed forefoot varus malalignment. What possible compensatory motion or posture might occur?
A. Excessive subtalar pronation.
B. Ipsilateral pelvic external rotation.
C. Hallux varus.
D. Genu recurvatum.

A

Answer: A
Possible compensatory motions or postures for forefoot varus malalignment include excessive midtarsal or subtalar pronation or prolonged pronation; plantarflexed first ray; hallux valgus; or excessive tibial; tibial and femoral; tibial, femoral and pelvic internal rotation; and/or with contralateral lumbar spine rotation.

How well did you know this?
1
Not at all
2
3
4
5
Perfectly
8
Q

A patient presents with fingertips that are rounded and bulbous. The nail plate is more convex than normal, These changes are the likely result of which condition?
A. Psoriasis.
B. Chronic hypoxia from heart diease.
C. Inflammation of the proximal and lateral nail folds.
D. Trauma to the nail bed.

A

Answer: B
Chronic hypoxia from heart disease or lung cancer and hepatic cirrhosis leads to clubbing of the fingers, characterized by fingertips that are rounded and bulbous and a nail plate that is more convex than normal.

How well did you know this?
1
Not at all
2
3
4
5
Perfectly
9
Q

The PT is examining a patient for right neck pain and spasms. Several inflamed submandibular nodes are noted. The nodes are approximately 1.0 cm in size, tender, and erythematous. The patient has no known history of cancer or metabolic diseases. What is the FIRST action the therapist should take?
A. Question the patient regarding impact of neck pain and emotional distress.
B. Question the patient regarding any recent dental or throat infections.
C. Have another therapist confirm the findings before implementing treatment.
D. Apply superficial heat and begin manual lymphatic drainage.

A

Answer: B
Typically, lymph nodes are not palpable, but they can become palpable in the presence of infection or metastases. Past medical history is instrumental to identifying when follow-up is required by the physician. Recent infections (especially dental or pharyngeal) can make the lymph nodes swollen, tender, erythematous, and/or firm. Medical referral is necessary.

How well did you know this?
1
Not at all
2
3
4
5
Perfectly
10
Q

10.A PT is reviewing a medical record prior to examining a patient for the first time. The suspected diagnosis is multiple sclerosis. On the neurologist’s note, the therapist finds the following: deep tendon reflex (DTR); right quadriceps is 2+, left quadriceps is 4+. What is the correct interpretation of these findings?
A. The right DTR is normal, the left is abnormal.
B. Both DTRs are abnormal and indicative of hyporeflexia.
C. The right DTR is exaggerated, the left is clearly abnormal.
D. Both DTRs are abnormal and indicative of upper motor neuron (UMN) syndrome.

A

Answer: A
DTRs are graded on a 1-4 scale. Scores include 0 (no response); 1+ (present but depressed; 2+ (normal); 3+ (increased, brisker than average; possibly but not necessarily); and 4+ (very brisk, hyperactive, with clonus, abnormal). In this case, the right DTR is normal; the left is abnormal and consistent with strong hypertonicity.

How well did you know this?
1
Not at all
2
3
4
5
Perfectly
11
Q

11.A patient has an episode of syncope in the physical therapy clinic. The therapist attempts to rule out orthostatic hypotension as the cause of the fainting. What is the BEST test protocol to use?
A. Palpate the carotid arteries and take resting HR and BP in the supine position.
B. Take resting HR and BP in supine, then in sitting, then in standing after 1 minute.
C. Take resting HR and BP in supine after 5 minutes, then in semi-Fowler position.
D. Take resting HR and BP in sitting and after 3 and 5 minutes of cycle ergometry exercise.

A

Answer: B
Orthostatic hypotension is a fall in BP with elevation of position; thus responses to movements (HR and BP) are tested from supine to sitting or sitting to standing. A small increase or no increase in HR upon standing may suggest baroreflex impairment. An exaggerated increase in HR upon standing may indicate volume depletion.

How well did you know this?
1
Not at all
2
3
4
5
Perfectly
12
Q

12.An impatient with a grade III diabetic foot ulcer is referred for physical therapy. Panafil has been applied to the necrotic tissue BID. The wound has no foul smell; however, the therapist notes a green tinge on the dressing. What is the BEST action for the therapist to take?
A. Fit the patient with a total contact cast.
B. Document the finding and contact the physician immediately.
C. Begin a trial of acetic acid to the wound.
D. Document the finding and continue with treatment.

A

Answer: D
In this case, the therapist should document the findings and continue with treatment. Panafil is a keratolvtic enzyme used for selective debridement. A greenish or yellowish exudate can be expected.

How well did you know this?
1
Not at all
2
3
4
5
Perfectly
13
Q

13.A PT receives a referral to examine the fall risk of an elderly patient with Parkinson’s disease who lives alone and has had two recent falls. Which activity is the MOST common reason for falls in the elderly?
A. Walking with a roller walker with hand brakes.
B. Climbing on a step stool to reach overhead objects
C. Turning around and sitting down in a chair.
D. Dressing while sitting on the edge of the bed.

A

Answer: C
Most falls occur during normal daily activity. Getting up or down from a bed or chair, turning, bending, walking, and climbing/descending stairs are all high-risk activities.

How well did you know this?
1
Not at all
2
3
4
5
Perfectly
14
Q

14.A patient with Addison’s disease is referred for physical therapy following a hip fracture.
Which of the following is a cardinal symptom of Addison’s disease?
A. Weight gain
B. Tremors
C. Asthenia
D. Diarrhea

A

Answer: C
The cardinal symptom of Addison’s disease is asthenia. The weakness is slowly progressive and debilitating.

How well did you know this?
1
Not at all
2
3
4
5
Perfectly
15
Q

15.A2-month-old child with bilateral hip dislocations is being discharged from an acute pediatric facility. The PT has developed a home exercise program and now needs to instruct the patients. What is the MOST important item for the therapist to assess before instructing the parents?
A. Their degree of anxiety and attention.
B. Their knowledge of the etiology of the hip dislocations.
C. The home environment.
D. The financial reimbursement plan.

A

Answer: A
A needs assessment should include a determination of the level of anxiety and ability to attend to the instructions given, If anxiety is high and the parents are unable to attend to the therapist’s instructions, risk of failure to perform the home exercises correctly is high.

How well did you know this?
1
Not at all
2
3
4
5
Perfectly
16
Q

16.A PT receives a referral for a young child that had been swung around while being held from the wrists. The referral reads, “functional disuse following nursemaid’s elbow.” Which of the following commonly results from a forceful longitudinal pull of the forearm of a child?
A. Superior subluxation of the radial head from the annular ligament.
B. Inferior subluxation of the ulna from the annular ligament.
C. Superior subluxation of the ulna from the annular ligament
D. Inferior subluxation of the radial head from the annular ligament.

A

Answer: D
The inferior subluxation of the radial head from the annular ligament typically occurs with a forceful longitudinal pull of the forearm in a child. It is also known as “baby sitter’s elbow.

How well did you know this?
1
Not at all
2
3
4
5
Perfectly
17
Q

17.Which is typical early clinical manifestation of cystic fibrosis (CF)?
A. Increase in secretions of the endocrine system.
B. Frequent recurrent urinary tract infections.
C. Excessive appetite and weight loss.
D. Increased FEV: (forced expiratory volume in 1 sec) during pulmonary function testing.

A

Answer: C
CF is an inherited disorder affecting the exocrine glands of the hepatic, digestive, and respiratory systems. The patient with CF is prone to chronic bacterial airway infections and progressive loss of pulmonary function from progressive obstructive lung disease. Early clinical manifestations include an inability to gain weight despite an excessive appetite and adequate caloric intake.

How well did you know this?
1
Not at all
2
3
4
5
Perfectly
18
Q

Which of the following activities demonstrates an infant’s integration of the asymmetrical tonic neck reflex?
A. Turns head to one side and brings opposite hand to mouth.
B. Can turn head to either side with extended arms.
C. Turns head to one side and brings hand to mouth on the same side.
D. Turns head to one side and looks at the extended arm on that side.

A

Answer: C
ATNR causes extension of upper extremity on the side the head is turned toward. Bringing the hand to the mouth would not be possible with an obligatory reflex.

How well did you know this?
1
Not at all
2
3
4
5
Perfectly
19
Q

19.A Physical therapy aide is cleaning a mat table with a new product supplied by the housekeeping department. The spray from the cleaning agent contact the skin resulting in irritation, redness, and some swelling. The symptoms were minor and abated within 20 minutes. Later, the aide informed the physical therapy supervisor of this situation. What action should the supervisor take?
A. Initiate first aid by rinsing the affected area with a skin cleanser and applying cortisone cream for the inflammation.
B. Inform housekeeping to immediately cease using the cleaning agent.
C. Fill out an incident/occurrence report and have the aide examined by employee health or their own primary care physician.
D. Fill out an incident/occurrence report and review the Material Safety Data Sheet (MSDS) from occupational Safety and Health Administration (OSHA) with the aide on how to properly handle the cleaning agent.

A

Answer: D
Material Safety Data Sheets are mandated by OSHA of the U.S Department of Labor, Thses sheets give employees information about potentially hazardous materials in the workplace and how to protect themselves. An incident/occurrence report is used to document situations that involve patients or staff that could have resulted in potential long-lasting or permanent harm and are part of an internal quality improvement program. The use of this cleaning agent may have some specific directions in method of application and precautions needed to be observed by the user. The MSDS would make this clear.

How well did you know this?
1
Not at all
2
3
4
5
Perfectly
20
Q

20.During surgery an apical lung tumor, the long thoracic nerve was injured. Muscle testing of the serratus anterior demonstrates its strength to be 3+/5. What is the BEST initial exercise for this patient?
A. Standing wall push-ups.
B. Standing arm overhead lifts using hand weights.
C. Supine arm overhead lifts using weights.
D. Sitting arm overhead lifts using a pulley.

A

Answer: A
The long thoracic nerve supplies serratus anterior muscle. With a muscle grade of 3+/5, the patient can then begin functional strengthening using standing wall push-ups, with resistance (provided by the patient’s own body.

How well did you know this?
1
Not at all
2
3
4
5
Perfectly
21
Q

21.A PT examination reveals posterior superior iliac spine (PSIS) is low on the left; anterior superior iliac spine (ASIS is high on the left; standing flexion test shows that the left PSIS moves first and farthest superiorly; Gillet’s test demonstrates that the left PSIS moves interiorly and laterally less than right; long sitting test shows that the left malleolus moves short to long; and the sitting flexion test is negative. Based on these findings, what is the therapist’s diagnosis?
A. Left upslip
B. Iliac inflare on the left
C. Left posterior rotated innominate
D. Left anterior rotated innominate

A

Answer: C
A posterior rotated innominate is a unilateral iliosacral dysfunction. The question outlines positive physical findings, both static and dynamic, found with this dysfunction. One of these positive findings alone does not confirm the diagnosis of left rotated posterior innominate.

How well did you know this?
1
Not at all
2
3
4
5
Perfectly
22
Q

22.A patient recovering from a burn on the back of the hand is referred to physical therapy for mobilization exercises. The therapist observes a 14-em irregular area that is thick and pink.
How should the therapist document this finding?
A. Hypertrophic scarring
B. An excoriation
C. Atrophic scarring
D. A scale

A

Answer: A
Hypertrophic scars are thick (raised) and pink (or red).

How well did you know this?
1
Not at all
2
3
4
5
Perfectly
23
Q

23.A patient presents with partial-and full-thickness burns on the chest and neck regions, The therapist decides to apply transcutaneous electrical nerve stimulation (TENS) before debridement to modulate pain. Which TENS mode should provide the BEST relief?
A. Acupuncture-like (low-rate) TENS
B. Brief intense TENS
C. Modulated TENS
D. Conventional (high-rate) TENS

A

Answer: B
Brief intense TENS is used to provide rapid-onset, short-term relief during painful procedures.
The pulse rate and pulse duration are similar to conventional TENS; however, the current intensity is increased to the patient’s tolerance.

How well did you know this?
1
Not at all
2
3
4
5
Perfectly
24
Q

24.A patient recovering from traumatic brain injury (TBI) demonstrates difficulties in feeding resulting from an unstable posture while sitting, The therapist determines that modification is necessary to ensure optimal function. What is the first body segment or segments that the therapist should align?
A. Trunk
B. Pelvis
C. Head
D. Lower extremities

A

Answer: B
Modification of the pelvic position in a neutral posture promotes good lumbar and trunk alignment. Many postural problems are correctable by aligning the pelvis first and achieving a stable base.

How well did you know this?
1
Not at all
2
3
4
5
Perfectly
25
Q

25.A patient demonstrates some out-of-synergy movements in the right upper extremity indicative of stage 4 recovery after a left cerebrovascular accident (CA). Which proprioceptive neuromuscular facilitation (PNF) pattern represents the BEST choice to promote continued recovery of the right upper extremity?
A. Bilateral symmetrical D2F and D2E, elbows straight
B. Chop, reverse chop with right arm leading
C. Lift, reverse lift with right arm leading
D. Bilateral symmetrical D1 thrust and reverse thrust

A

Answer: B
Both chop and reverse chop patterns move the affected arm out-of-synergy

How well did you know this?
1
Not at all
2
3
4
5
Perfectly
26
Q

26.An elderly individual has limited endurance as a result of a sedentary lifestyle. There is no history of cardiorespiratory problems. An exercise tolerance test was negative for coronary heart disease. What is the BEST initial exercise prescription for this individual?
A. 40%-50% HR max
B. 35%-50% of VO2max
C. 60%-90% HR max
D. 30%-50% HRmax

A

Answer: C
An appropriate initial exercise prescription for an asymptomatic elderly individual with general deconditioning is 60%-90% of HRmax which is equivalent to 50%-85% of HR reserve (Karvonen’s formula). This is within the established intensity guidelines for adults for aerobic exercise training. Duration should be discontinuous, and exercise should be performed most days of the week (ACSM guidelines for Exercise Testing and Prescription, 8’ ed.).

How well did you know this?
1
Not at all
2
3
4
5
Perfectly
27
Q

27.A patient is admitted to a coronary care unit with a mild myocardial infarction (MI). after 2 days, the patient is referred to physical therapy for inpatient cardiac rehabilitation. During an initial exercise session on the unit, the patient reports chest pain, appears anxious and wants to go back to bed to rest. What is the therapist’s BEST initial course of action?
A. Assist the patient back to bed and contact the charge nurse on the floor.
B. Sit the patient and monitor vital signs carefully during the rest period.
C. Assign the PTA to assist the patient back to bed and monitor vital signs carefully
D. Terminate the exercise and contact the attending physician immediately

A

Answer: B
If the chest pain (angina) is exercise induced, this is an indication to stop the exercise session and provide a period of rest during which time the patient is closely monitored. Recovery is expected after a period of rest.

How well did you know this?
1
Not at all
2
3
4
5
Perfectly
28
Q

28.A patient with a 10-year history of sIcleroderma is referred for physical therapy to improve functional status and endurance. The patient was recently treated with corticosteroids for a bout of myositis. Examination findings reveal limited ROM and fibrotic soft tissue along with hyperesthesia. What is the BEST choice for initial intervention?
A. Treadmill walking using body weight support at an intensity of 40% HRmax
B. Active range of motion (AROM) exercises and walking in a therapeutic pool
C. Closed-chain and modified aerobic step exercises
D. Soft tissue mobilization and stretching

A

Answer: B
Scleroderma (progressive systemic sclerosis) is a chronic, diffuse disease of connective tissues causing fibrosis of skin, joints, blood vessels, and internal organs. Patient typically demonstrate symmetrical skin thickening and visceral involvement of the gastrointestinal tract, lungs, heart, and kidneys along with hypersensitivity to touch. The BEST choice for initial intervention is to exercise in the pool. The warmth and buoyancy of the water will enhance the patient’s movements and decrease pain.

How well did you know this?
1
Not at all
2
3
4
5
Perfectly
29
Q

29.A patient is referred to physical therapy after a fall injury (fractured left hip with operative reduction, internal fixation [ORIF]). Medical history reveals a diagnosis of mild Alzheimer’s type dementia (AD). Given this stage of AD, which change should not be evident in this patient?
A. Profound communication deficits
B. Memory loss
C. Anxiety and irritability
D. Difficulty concentrating

A

Answer: A
Profound communication deficits (inability to speak), global deterioration of mental functions (delusions, hallucinations, fragmented memory), agitation, and pacing (sundowning) are all characteristic of later stages of AD.

How well did you know this?
1
Not at all
2
3
4
5
Perfectly
30
Q

30.A patient recovering from stroke with minimal lower extremity weakness and spasticity is able to walk without an assistive device. The therapist observes that as the patient walks there is noticeable hip hiking on the more affected side during swing phase. What is the BEST initial intervention?
A. Bridging exercises progressing to sit-to-stand training.
B. Marching while sitting on a therapy ball.
C. Standing and marching with manual pressure applied downward on the pelvis
D. Partial wall squats using a small ball held between the knees.

A

Answer: B
Hip hiking is a compensatory response for weak hip and knee flexors or extensor spasticity.
Active exercises for the hip and knee flexors (marching) is the most appropriate intervention.

How well did you know this?
1
Not at all
2
3
4
5
Perfectly
31
Q

The therapist is instructing a patient with traumatic brain injury (TBI) how to lock the brakes on a wheelchair. The patient is right-handed, and the right upper extremity is more affected than the left. What is the BEST motor learning strategy to use with this patient?
A. Have the patient practice brake locking using the left hand to assist the right.
B. Guide the patient’s right hand through the locking motions, then the left.
C. Verbally talk the patient through the locking motions, practicing with both hands simultaneously.
D. Have the patient practice locking the brakes first with the left hand and then with the right.

A

Answer: D
Using the motor learning strategy of transfer of training is best to use with this patient. Practice is performed with the less affected extremity first and then progressed to use of the more affected extremity.

How well did you know this?
1
Not at all
2
3
4
5
Perfectly
32
Q

32.In neural tension testing, what position will BEST bias the tibial nerve?
A. Straight leg raise with plantarflexion and eversion
B. Straight leg raise with dorsiflexion and inversion
C. Straight leg raise with plantarflexion and inversion
D. Straight leg raise with dorsiflexion and eversion

A

Answer: D
A straight leg raise with dorsiflexion and eversion will best bias the tibial nerve. This is the optimal position for neural tissue provocation of the tibial nerve. Neural tension techniques are used to decreaseadverse mechanical tension on the nerves. Peripheral nerves can often become trapped within the tissues, where there can be a pull on the nerve with movement. This technique frees up the nerve so that it can slide in its sheath.

How well did you know this?
1
Not at all
2
3
4
5
Perfectly
33
Q

33.A therapist is examining a patient with chronic obstructive pulmonary disease (COPD)
GOLD stage III. What would be a clinical finding that the therapist would expect for this patient?
A. Decreased anteroposterior to lateral chest ratio.
B. Muscle wasting.
C. Use of supplemental oxygen.
D. Weight gain.

A

Answer: B
Muscle wasting is a common manifestation of COPD. The cause of muscle wasting is not clear, but it is not simply a malnutrition problem. The results of this muscle wasting are peripheral weakness, impaired functional abilities, poor quality of life, and a poor prognostic sign.

How well did you know this?
1
Not at all
2
3
4
5
Perfectly
34
Q

34.What muscle length test for the tensor fascia lat is recommended in a patient with decreased muscle length of the rectus temoris?
A. Modified Ober test (knee extended).
B. FAIR (flexion, adduction, internal rotation) test.
C. Ober test (knee flexed).
D. Ely’s test.

A

Answer: A
The modified Ober test is performed with the knee extended, which allows the patient to attain the necessary hip extension required to assess the tensor fascia lata.

How well did you know this?
1
Not at all
2
3
4
5
Perfectly
35
Q

35.During a postural screen for chronic shoulder pain, the therapist observes excessive internal rotation of the shoulders and winging of the scapula during overhead motion, What is the BEST choice for exercise intervention?
A. Strengthening of rhomboids and stretching of upper trapezius.
B. Strengthening of pectoral muscles and stretching of upper trapezius.
C. Strengthening of upper trapezius and stretching of pectoral muscles.
D. Strengthening of middle and lower trapezius and stretching of pectoral muscles.

A

Answer: D
Abnormal posture that produces excessive internal rotation of the shoulders may result in chronic shoulder impingement syndrome due to a loss of scapular stability with overhead motion. Shoulder pain is likely to continue until a balance between anterior and posterior trunk musculature is achieved. The anterior chest muscles (pectorals) are shortened and need stretching and posterior trunk muscles (middle and lower trapezius) are stretched and need strengthening.

How well did you know this?
1
Not at all
2
3
4
5
Perfectly
36
Q

36.Examination of a patient with a dermal ulcer over the coccyx reveals a wound exposing the deep fascia. There is no necrotic tissue, exudate is minimal, and the borders of the ulcer are diffusely covered with granulation tissue. Previous treatment has included wet-to-dry dressings with normal saline. What is the BEST choice for intervention?
A. Hydrogel dressings and whirlpool immersion
B. Continuation of the same treatment
C. Wound irrigation with pressures below 15 psi
D. Calcium alginate dressings

A

Answer: C
Low -pressure wound irrigation helps to decrease colonization and prevent infection.

How well did you know this?
1
Not at all
2
3
4
5
Perfectly
37
Q

37.Based on the spinal defect shown in the diagram, what lumbar spinal motion should be avoided?
A. Rotation
B. Extension
C. Lateral flexion
D. Flexion

A

Answer: B
With spondylolisthesis, there is typically an anterior slippage of one vertebra on the vertebra below. Because of the anterior shearing forces acting at the vertebra caused by the wedge shape of the vertebra and gravity, spinal extension positions should be avoided.

How well did you know this?
1
Not at all
2
3
4
5
Perfectly
38
Q

38.A computer specialist is unable to work because of weakness and altered sensation in the dominant right hand. The patient complains of pain and tingling of the thumb, index finger, long finger, and radial half of the ring finger. The therapist observes thenar weakness and atrophy. Strength, reflexes, and sensation are within normal limits throughout the remainder of the right upper extremity. These signs and symptoms are characteristic of what diagnosis?
A. Ulnar nerve compression
B. Carpal tunnel syndrome
C. Pronator teres syndrome
D. Cervical root compression

A

Answer: B
The pattern of motor and sensory loss corresponds to the median nerve distribution in the hand.
The most likely cause is carpal tunnel syndrome.

How well did you know this?
1
Not at all
2
3
4
5
Perfectly
39
Q

39.A patient presents with rapidly progressive symmetrical weakness that started in the distal lower extremity muscles but now has ascended to include proximal trunk and upper extremity muscles. The motor segments of the lower cranial nerves are also showing impairment. The patient complains of abnormal sensations of tingling and burning of the affected extremities. Consciousness, cognition, and communication are all normal. These signs and symptoms are characteristic of what diagnosis?
A. Multiple sclerosis
B. Guillain-Barre syndrome
C. Amyotrohic lateral sclerosis
D. Postpolio syndrome.

A

Answer: B
These signs and symptoms are characteristic of Guillain-Barre syndrome, a peripheral neuropathy in which there is inflammation and demyelination of peripheral motor and sensory nerve fibers, Early in its progression, either upper or lower motor signs may predominate.

How well did you know this?
1
Not at all
2
3
4
5
Perfectly
40
Q

40.A patient recovering from surgery for triple coronary artery bypass grafts is scheduled to begin a phase III cardiac rehabilitation program. During the resistance training portion of the circuit training program, the therapist instructs the patient to avoid the Valsalva maneuver.
What are the expected adverse effects of the Valsalva maneuver?
A. Slowing of pulse and increased venous pressure are possible.
B. The decreased return of blood to the heart can lead to pitting edema.
C. Heart rate (HR) and blood pressure are likely to be elevated.
D. A cholinergic or vagal response can occur.

A

Answer: A
The Valsalva maneuver results from forcible exhalation with the glottis, nose, and mouth closed. It increases intrathoracic pressures and causes slowing of the pulse, decreased return of blood to the heart, and increased venous pressure. Although Valsalvas occur during normal daily activities (breath holding, straining), they can be dangerous for patients with cardiovascular disease. On relation, blood rushes to the heart and can overload the cardiac muscle, resulting in cardiac arrest:

How well did you know this?
1
Not at all
2
3
4
5
Perfectly
41
Q

41.A patient experiences color changes in the skin during position changes of the foot. During evaluation, pallor develops. When the limb is then positioned in the seated hanging position, hyperemia develops. What do these changes indicate?
A. Lymphedema
B. Arterial insufficiency
C. Deep vein thrombophlebitis
D. Chronic venous insufficiency

A

Answer: B
Arterial insufficiency can be determined by skin color changes during position changes of the foot (termed rubor of dependency test).

How well did you know this?
1
Not at all
2
3
4
5
Perfectly
42
Q

42.An elderly and frail adult is referred to physical therapy for an examination of balance. The patient has a recent history of falls (two in the last 6 months). Based on knowledge of balance changes in the elderly and scoring of standardized balance measures, which of the following test results BEST indicates increased fall risk?
A. Tinetti Performance Oriented Mobility Assessment (POMA) score of 27
B. Functional Reach of inches
C. Timed Get UP & GO (GUG) test result of 13 seconds.
D. Berg Balance score of 50

A

Answer: B
All of these instruments can be used to examine functional balance and fall risk. A functional
Reach score of <10 is indicative of increased fall risk.

How well did you know this?
1
Not at all
2
3
4
5
Perfectly
43
Q

43.During an examination, a patient complains of right upper quadrant pain and tenderness. The PT percusses over the costal margin at the point where the lateral border of the rectus muscle intersects with the costal margin. The patient complains of acute pain and stops inspiratory effort. What does this patient’s response indicate?
A. Hernia
B. Acute cholecystitis
C. Irritation of the psoas muscle by an inflamed appendex
D. Peritoneal inflammation

A

Answer: B
Percussion for costovertebral tenderness that reveals a sharp increase in tenderness with a sudden stop in inspiratory effort is a positive Murphy’s sign and is indicative of acute cholecystitis.

How well did you know this?
1
Not at all
2
3
4
5
Perfectly
44
Q

44.A 72-year-old patient is walking on a treadmill in the physical therapy department while vital signs and pulse oximetry are being monitored. It is noted that the patient’s arterial oxygen saturation (SpOz) drops from 95% to 92%. What is the therapist’s BEST response to this change?
A. Place a 40% Oz face mask on the patient for the remainder of the exercise session.
B. Not use supplemental Oz
C. Place 2 L of Oz by nasal cannula on the patient for the remainder of the exercise session
D. Place a 100% O2 face mask on the patient for the remainder of the exercise session.

A

Answer: B
A 72-year-old patient would likely have a resting SpO2 of 95% from the changes associated wit aging alon. There is no need to supplement oxygen in this case.

How well did you know this?
1
Not at all
2
3
4
5
Perfectly
45
Q

45.The therapist is evaluating the needs of a young child who is diagnosed with myelodysplasia at the T10 level. What is the therapist’s BEST choice of mobility device for this child to use in the school environment?
A. Bilateral knee-ankle-foot orthosis (KAFO)
B. Parapodium
C. Bilateral hip-knee-ankle-foot orthosis (HKAFO)
D. Lightweight wheelchair

A

Answer: D
The lightweight wheelchair is the MOST beneficial choice for this child. It provides effective and efficient mobility.

How well did you know this?
1
Not at all
2
3
4
5
Perfectly
46
Q

46.A patient has limited right rotation caused by left thoracic facet joint capsular tightness at
T6-7, what arthrokinematic glide would MOST effectively improve right rotation in sitting?
A. Superior and anterior glide on the right T7 transverse process.
B. Superior and anterior glide on the left T7 transverse process.
C. Superior and anterior glide on the right T6 transverse process.
D. Superior and anterior glide on the left T6 transverse process.

A

Answer: D
Because the left thoracic facet joint capsule is restricting movement, motion that would stretch the capsule would facilitate improved right rotation. With right rotation, the left superior facets move upward (opening the joint and stretching the capsule) and the right facets move downward (closing the joint and putting the capsule on relative slack).

How well did you know this?
1
Not at all
2
3
4
5
Perfectly
47
Q

47.Which muscles provide fairly continuous activity during quiet standing when measure by
EMG?
A. Quadriceps femoris and anterior tibialis
B. Posterior tibialis and intrinsic foot muscles
C. Anterior tibialis and peroneals
D. Soleus and gastrocnemius

A

Answer: D
The soles and gastrocnemius muscles oppose the dorsiflexion moment that exists at the ankle as a result of the line of gravity, which falls slightly anterior to the lateral malleolus. This fairly continuous activity is crucial for maintaining balance during quiet standing.

How well did you know this?
1
Not at all
2
3
4
5
Perfectly
48
Q

48.A patient falls while walking in the parallel bars. The therapist is required to fill out an incident report of te event. In addition to the names of those involved, what information is required in an incident report?
A. A description of the event, where the patient was injured, and the corrective actions to be taken.
B. Witness reports and therapist’s opinion as to the cause
C. What occurred, when and where it occurred, and witness statements.
D. The cause of this fall and cross-references to others who gave fallen in the parallel bars

A

Answer: C
The typical information included on an incident report are the names of those involved, inclusive of witnesses, what occurred, when it occurred, and where it occurred.

How well did you know this?
1
Not at all
2
3
4
5
Perfectly
49
Q

49.A therapist is beginning manual lymphatic drainage for a patient recently diagnosed with secondary lymphedema in the left upper extremity following a radical mastectomy. What is the BEST choice for initial bandaging of the limb?
A. Long-stretch compression wrap (Ace wrap).
B. Custom-made low-elastic garment
C. Gauze wrap
D. Short-stretch compression wrap (comprilan®)

A

Answer: D
A short-stretch wrap has a low resting pressure and high working pressure. This means it has enough pressure to enhance lymphatic return at rest, improve the activity of the lymphangion (contractile unit of the lymphatic system), and facilitate increased return during muscle pumping activities.

How well did you know this?
1
Not at all
2
3
4
5
Perfectly
50
Q

50.An elderly individual was found unconscious at home and was hospitalized with a diagnosis of cerebrovascular accident (CVA). Examination by the PT reveals normal sensation and movement on the right side of the body with impaired sensation (touch, pressure, proprioception) and paralysis on the left side of the body. The left side of the lower face and trunk are similarly impaired. What is the MOST likely location of the CVA?
A. Left parietal lobe
B. Right parietal lobe
C. Left side of brain stem
D. Spinal cord

A

Answer: B
This patient demonstrates involvement of the long tract (sensory and motor) indicative of involvement of the contralateral cerebral cortex, parietal lobe.

How well did you know this?
1
Not at all
2
3
4
5
Perfectly
51
Q

51.The loss of sensory function in peripheral neuropathy is often among the first noticeable symptoms. With large fiber damage, what is the typical pattern of the sensory loss?
A. Allodynia of the feet accompanied by pronounced dorsiflexor weakness
B. Band-like dysesthesias and paresthesias in the hips and thighs
C. Paresthesias affecting primarily the proximal limb segments and trunk
D. Stocking and glove loss of light touch and position sense

A

Answer: D
Symmetrical involvement of sensory fibers, progressing from distal to proximal, is the hallmark of polyneuropathy. It is termed “stocking and glove distribution” and is the result of the dying back of the longest fibers in all the nerves from distal to proximal. Sensory symptoms include decreased sensation and pain, paresthesias, and dysesthesias (abnormal sensations such as numbness, tingling, or prickling).

How well did you know this?
1
Not at all
2
3
4
5
Perfectly
52
Q

52.The PT is supervising a phase II cardiac rehabilitation class of 10 patients. One of the patients, who is being monitored with radiotelemetry, is having difficulty. Which change would be a criterion for terminating this exercise session?
A. An increase in systolic BP to 150 and diastolic BP to 90
B. 1-mm ST segment depression, upsloping.
C. A second-degree atrioventricular (AV) heart block
D. An increase in HR 20 beats/minute above resting

A

Answer: C
Criteria for reducing exercise intensity or termination according to the American College of Sports Medicine include (1) onset of angina and other symptoms of exertional intolerance; (2) systolic BP≥ 240 mm Hg, diastolic BP ≥ 110 mm Hg; (3) > 1-mm ST segment depression, horizontal or downsloping; (4) increased frequency of ventricular arrhythmias; and (5) second-degree or third-degree AV block or other significant electrocardiogram (ECG) disturbances.

How well did you know this?
1
Not at all
2
3
4
5
Perfectly
53
Q

53.A patient presents with severe claudication that is evident when walking distances greater than 200 feet. The patient also exhibits muscle fatigue and cramping of both calf muscles.
Upon examination, the PT finds the skin is pale and shiny with some trophic nail changes.
What is the BEST choice for this patient’s initial exercise program?
A. Avoid any exercise stress until the patient has been on calcium channel blockers for at least 2 weeks.
B. Begin with an interval walking program, exercising just to the point of pain.
C. Utilize non-weight-bearing exercises such as cycle ergometry.
D. Utilize a walking program of moderate intensity, instructing the patient that some pain is expected and to be tolerated.

A

Answer: B
This patient is exhibiting classic signs of peripheral artery disease (PAD). Rehabilitation guideline for arterial disease include using an intermittent walking program of moderate intensity and duration, 2-3 times/day, 3-5 days/wk. the patient should be instructed to exercise to the point of claudication pain within 3-5 minutes, not beyond.

How well did you know this?
1
Not at all
2
3
4
5
Perfectly
54
Q

54.A patient is hospitalized with diabetes and a large stage I plantar ulcer located over the right heel. The patient has been non-weight bearing for the past 2 weeks as a result of the ulcer.
What is the BEST choice for this patient’s initial intervention?
A. Wash the foot and apply skin lubricants followed by a transparent film dressing.
B. Clean and bandage with a sterile gauze dressing.
C. Refer the patient for a surgical consult
D. Clean and debride the wound and apply a hydrogel dressing.

A

Answer: D
A stage II ulcer (deep ulcer) involves a partial-thickness skin with involvement of epidermis, dermis, or both; it is reversible. Intervention should be directed toward improving perfusion and relieving localized pressure. The wound should be cleaned with an antimicrobial agent, debrided of necrotic tissue, and covered with a sterile dressing. Hydrogel dressings maintain moisture in the wound bed, soften necrotic tissue, and support autolytic debridement. Pressure relief is also an important consideration. Techniques of protective foot care should be taught.

How well did you know this?
1
Not at all
2
3
4
5
Perfectly
55
Q

55.A PT is prescribing a wheelchair for a patient with left hemiplegia who is of average height (5 feet 7 inches). Which of the following is the BEST choice to improve the patient’s function?
A. Desk armrests
B. A 20-inch seat height
C. A 17.5-inch seat height
D. Elevating leg rests.

A

Answer: C
A hemi-or low-seat wheelchair has a seat height of 17.5 inches. The lower seat height permits the patient to propel and steer the wheelchair using the sound right upper and lower extremities.

How well did you know this?
1
Not at all
2
3
4
5
Perfectly
56
Q

A patient who is participating in a cardiac rehabilitation program suddenly collapses and falls to the floor. The PT is the lone rescuer on site. The therapist checks for a response and finds the patient unresponsive. After activating the emergency response system (phone 911), what is the BEST action for the therapist to take?
A. Use the automated external defibrillator (AED to shock the patient after 3 minutes of cardiopulmonary resuscitation (CPR)
B. Begin CPR and attach and use the AED as soon as possible
C. Give 100 chest compressions per minute.
D. Give two rescue breaths followed by 15 chest compressions, repeating the cycle for at least 2 minutes.

A

Answer: B
guideline from the American Heart Association (2010) concerning Basic Life Support and CPR specify that the first responder call 911 for unresponsive adults, get an AED (if available), and return to the victim to provide CPR and defibrillation, if needed. Trained HCs can use ventilations (1 breath every 8 seconds) with chest compressions (at least 100/minute.

How well did you know this?
1
Not at all
2
3
4
5
Perfectly
57
Q

57.A patient who was caster for 3 weeks after a grade III right ankle sprain has been referred to physical therapy for mobility exercises. Examination shows a loss of 10° of dorsiflexion.
Which activity will be the MOST difficult for the patient?
A. Ambulating over rough surfaces
B. Descending stairs
C. Ambulating barefoot
D. Descending a ramp

A

Answer: B
Loss of dorsiflexion will make descending stairs most difficult because the ankle must have dorsiflexon during the single-limb support phase during descent.

How well did you know this?
1
Not at all
2
3
4
5
Perfectly
58
Q

What is the torque output produced in the sitting position during isokinetic exercise involving the hamstring muscle?
A. Higher than the torque actually generated by the contracting hamstrings.
B. Lower because of resistance of the quadriceps.
C. Higher because of eccentric assistance of the quadriceps.
D. Lower than the torque actually generated by the hamstrings

A

Answer: A
Gravity-produced torque adds to the force generated by the hamstrings when they contract, giving a higher torque output than is actually produced by the muscle (gravity-assisted exercise). Testing values may be misleading; software is available to correct for the effects of gravity.

How well did you know this?
1
Not at all
2
3
4
5
Perfectly
59
Q

59.Which is the BEST choice of manual therapy technique to correct a closing restriction of T5 on T6?
A. Unilateral posteroanterior (PA) pressure at a 60° angle on the left transverse process of
T6 while stabilizing T5.
B. Central PA pressure at a 60° angle on the spinous process of T6 while stabilizing T5
C. Unilateral PA pressure at a 45° angle on the right transverse process of T6 while stabilizing T5
D. Central PA pressure at a 45° angle on the spinous process of T5 while stabilizing T6

A

Answer: B
In a closing restriction, the inferior facets of the superior vertebrae will not inferiorly glide on the superior facets of the inferior vertebra. Therefore, T5 inferior facets will not caudally glide on the superior facets of 6. Stabilizing T5 and application of pressure to T6 localizes the cephalad movement of the superior facets T6 on T5 bilaterally. The angle of the thoracic facets is 60°; therefore, the application of force should be at the same plane.

How well did you know this?
1
Not at all
2
3
4
5
Perfectly
60
Q

60.A patient presents with an acute onset of vertigo overnight. Symptoms worsen with rapid change in head position. If the head is held still, symptoms subside usually within 30-60 seconds. What is the MOST likely cause of these symptoms?
A. Meniere’s disease
B. Benign paroxysmal positional vertigo (BPPV)
C. Bilateral vestibular neuritis
D. Acoustic neuroma

A

Answer: B
BPPV is characterized by acute onset of vertigo and is positional, related to the provoking stimulus of head movement.

How well did you know this?
1
Not at all
2
3
4
5
Perfectly
61
Q

61.A patient with a significant history of coronary artery disease is currently taking atropine.
Based on knowledge of this medication, what are the expected effects?
A. Increased HR and contractility at rest.
B. Increased myocardial ischemia.
C. Palpitations at rest and with exercise.
D. Orthostatic hypotension.

A

Answer: A
Atropine is an anticholinergic agent (it blocks the action of acetylcholine at parasympathetic sites in smooth muscle, secretory glands, and the central nervous system). It produces an increase in HR and contractility and is used to treat symptomatic sinus bradycardia and exercise-induced bronchospasm.

How well did you know this?
1
Not at all
2
3
4
5
Perfectly
62
Q

62.A patient recovering from stroke is taking warfarin (Coumadin). What potential adverse reactions are associated with this medication?
A. Hematuria and ecchymosis
B. Palpitations and edema
C. Edema and dermatitis
D. Cellulitis and xeroderma.

A

Answer: A
Warfarin sodium (Coumadin) is an anticoagulant indicated in the prophylaxis and treatment for venous trombosis, pulmonary embolism, and thromboembolic disorders. Potential adverse reactions include hematuria and ecchymosis (skin discoloration and hemorrhaging). Serious bleeding is possible with drug toxicity.

How well did you know this?
1
Not at all
2
3
4
5
Perfectly
63
Q

63.The PT review the laboratory results of a patient admitted to the acute care hospital
yesterday: Hematocrit 45%, fasting blood glucose 180 mg/dL, and cholesterol 180 mg/dL.
Based on these laboratory results, what condition is MOST likely?
A. Polycythemia vera
B. Hyperglycemia of diabetes
C. Anemia
D. Hyperlipidemia

A

Answer: B
Normal fasting blood glucose for adults is 65-99 mg/dL. A fasting blood glucose level of 180mg/dL is abnormal and indicative of hyperglycemia of diabetes.

How well did you know this?
1
Not at all
2
3
4
5
Perfectly
64
Q

64.A patient recovering from a stroke is having difficulty with stair climbing. During ascent, the patient is able to position the more involved foot on the step above but is unable to transfer the weight up to the next stair level. What is the BEST exercise intervention to remediate this problem?
A. Bridging, holding.
B. Standing, side steps.
C. Standing, partial wall squats
D. Plantigrade, knee flexion with hip extension

A

Answer: C
The quadriceps muscle is responsible for most of the energy generation needed to transfer up stairs to the next level. Partial wall squats are the BEST choice to strengthen these muscles (closed-chain exercise). During forward continuance (corresponding to mid-stance), the ankle plantarflexors assist. Hip extensors are also active concentrically, assisting these actions.

How well did you know this?
1
Not at all
2
3
4
5
Perfectly
65
Q

65.An elderly patient has been hospitalized and on complete bed rest for 10 days. A physical therapy referral requests mobilization out of bed and ambulation. The patient complains of aching in the right calf. The therapist’s examination reveals calf tenderness with slight swelling and warmth. What is the BEST course of action for the therapist?
A. Begin with ankle pump exercises in bed and then ambulate.
B. Postpone ambulation and report the findings immediately.
C. Ambulate the patient with support stockings on.
D. Use only AROM exercises with the patient sitting at the edge of the bed.

A

Answer: B
The patient is exhibiting early signs of acute deep vein thrombophlebitis (DVT). These findings should be reported immediately.

How well did you know this?
1
Not at all
2
3
4
5
Perfectly
66
Q

66.A 9-year-old boy with Duchenne’s muscular dystrophy is referred for home care. How should the therapist begin the examination?
A. Ask the parents to outline the boy’s past rehabilitation successes.
B. Ask the child and his parents to describe the boy’s most serious functional limitations
C. Perform a complete motor examination
D. Perform a functional examination using the weeFIM

A

Answer: B
The child and his parents/caretakers play an important part in determining impairments, functional limitations, disability, and future interventions. Taking a thorough initial history is important in determining what other components of the examination would be appropriate.

How well did you know this?
1
Not at all
2
3
4
5
Perfectly
67
Q

67.During an examination of an adolescent female who complains of anterior knee pain, the PT observes that the lower extremity shows medial femoral torsion and toeing-in position of the feet. What pathology of the hip is commonly associate with medial femoral torsion and toeing-in’?
A. Retroversion
B. Anteversion
C. Medial/internal rotation
D. Lateral/external rotation

A

Answer: B
The pathology commonly associated with medial femoral torsion and toeing-in is hip anteversion due to an increase in the antetorsion angle (>15°) between the femoral condyles and the neck of the femur.

How well did you know this?
1
Not at all
2
3
4
5
Perfectly
68
Q

68.A patient with osteoporosis and no fractures complains of increased middle and lower back pain during breathing and other functional activities. What is the MOST beneficial exercise intervention for this patient?
A. Trunk flexion and rotation exercises
B. Trunk flexion and extension exercises
C. Trunk extension and abdominal stabilization exercises
D. Trunk rotation and abdominal stabilization exercises.

A

Answer: C
It is important to strengthen from the core to the floor as well as train in proprioception and balance enhancement techniques. Trunk extension and abdominal stabilization exercises are indicated.

How well did you know this?
1
Not at all
2
3
4
5
Perfectly
69
Q

69.A single 22-year-old woman who is 3 months pregnant arrives at a therapist’s private practice complaining of shoulder and leg pain. She has a black eye and some bruising at the wrists. The state in which the therapist practices has direct access. What is the BEST course of action for the therapist?
A. Examine the patient, and if abuse is suspected, report the findings to the appropriate authorities
B. Administer massage for bruising, TENS, and ice modalities for pain, as indicated by the examination findings
C. Direct the patient to the nearest ambulatory care center for physician evaluation
D. Refuse to examine the patient and send her to the nearest emergency room

A

Answer: A
According to the American Physical Therapy Association’s (APTA) Guideline for Recognizing and Providing Care for Victims of Domestic Violence, this patient falls into a category of high risk. Women between the ages of 17 and 28 years and women who are single, separated, or divorced or who are planning a separation or divorce are at high risk. Battered women usually have more than one injury. Most injuries occur in the head, face, neck, breasts, and abdomen.
According to the American Medical Association (AMA), battered women represent 23% of pregnant women who seek prenatal care. The victim may not volunteer information about her situation, but more often than not when asked will reveal it.
It is important for the PT to examine the patient and, if abuse is suspected, report the findings to appropriate authorities. The therapist should be familiar with resources available for victims of domestic violence and their own state reporting laws.

How well did you know this?
1
Not at all
2
3
4
5
Perfectly
70
Q

70.An elderly patient presents with stage III decubitus ulcer on the plantar surface of the right foot. After a series of conservative interventions with limited success, the therapist chooses to apply electrical stimulation for tissue repair. What is the BEST electrical current to administer in this case?
A. Medium-frequency burst current
B. High-volt monophasic pulsed current
C. Medium-frequency beat current.
D. Low-volt biphasic pulsed current.

A

Answer: B
Because high-volt pulsed current is a monophasic, unidirectional current, the unidirectional current would produce a therapeutic effect at the active (treatment) electrode. A negative charge (polarity) should be applied for a bactericidal effect or a positive charge given to promote wound healing.

How well did you know this?
1
Not at all
2
3
4
5
Perfectly
71
Q

71.A patient presents with pain radiating down the posterior hip and thigh as a result of a herniated disc in the lumbar spine. The therapist decides to apply mechanical traction. If the patient can tolerate it, what is the preferred patient positon?
A. Supine with one knee flexed
B. Prone with pillow under the abdomen
C. Supine with both knees flexed
D. Prone with no pillow

A

Answer: D
Placing the patient in the prone position would better align the spine so that the pull of the traction would be along the axis of the vertebral bodies.

How well did you know this?
1
Not at all
2
3
4
5
Perfectly
72
Q

72.A patient recovering from stroke demonstrates hemiparesis of the right upper extremity with moderate flexion and extension synergies (flexion stronger than extension). The therapist’s goal is to strengthen the shoulder muscles first to promote elevation of the arm. What is the BEST exercise intervention to achieve this goal?
A. Shoulder abduction with elbow flexion
B. Shoulder flexion with elbow extension
C. Shoulder horizontal adduction with elbow extension
D. Shoulder horizontal adduction with elbow flexion

A

Answer: B
Obligatory hemiplegic synergies are present and should not be reinforced. Shoulder flexion with elbow extension is the correct choice. It is an out-of-synergy combination that strengthens the shoulder flexors needed to stabilize the shoulder in an elevated functional position. In sitting, bending forward with elbow straight and hand touching floor is a good example of an early activity to promote this.

How well did you know this?
1
Not at all
2
3
4
5
Perfectly
73
Q

73.A physical therapist shines a light into a patient’s eye and observes the pupil of the eye.
Constriction of the pupil results. Which cranial nerve is being tested?
A. Abducens
B. Trochlear
C. Optic
D. Oculomotor

A

Answer: D
The pupillary reflex (constriction of the pupil) is a function of the efferent portion of the oculomotor nerve (CN III).

How well did you know this?
1
Not at all
2
3
4
5
Perfectly
74
Q

74.A patient is referred for physical therapy after a graded exercise test (GXT). The physician reports the test was positive and had to be terminated at 7 minutes. Which of the following criteria is an absolute indication for terminating exercise testing?
A. Mild angina and dyspnea with progressive increases in the treadmill speed and grade
B. A hypertensive response with blood pressure of at least 170/95.
C. ST segment depression from baseline of 3-mm horizontal or downsloping depression.
D. ECG changes from baseline of 1-mm ST segment elevation.

A

Answer: C
A positive GXT indicates myocardial ischemia with increasing exercise intensities. The optimal test duration is 8-12 minutes but can be terminated if symptoms of exertional intolerance are evident. The American College of Sports Medicine (ACSM) indicates these include ECG changes from baseline (> 2 mm horizontal or downsloping; ST segment depression, or > 2 mm ST segment elevation).

How well did you know this?
1
Not at all
2
3
4
5
Perfectly
75
Q

75.A college soccer player sustained a hyperextension knee injury when kicking the ball with the other lower extremity. The patient was taken to the emergency room of a local hospital and was diagnosed with “knee sprain.” The patient was sent to physical therapy the next day for rehabilitation. As part of the examination, the therapist conducts the test shown in the figure. Based on a positive test, what type of exercise intervention is indicated during the acute phase of treatment?
A. Open-chain terminal knee extension exercises.
B. Closed-chain terminal knee extension exercises.
C. Plyometric functional exercises.
D. Agility exercises.

A

Answer: B
The test that was conducted was a Lachman’s test to determine integrity of the ACL. A positive test suggests laxity of the ACL. Closed-chain-terminal knee extension exercises are safe and effective secondary to the dynamic stability inherent with this type of exercise.

How well did you know this?
1
Not at all
2
3
4
5
Perfectly
76
Q

76.A patient with a grade III diabetic ulcer is being treated with a calcium alginate wound dressing. What are the primary indications for this type of wound dressing?
A. Provide semirigid support for the limb while maintaining a sterile field.
B. Facilitate autolytic debridement and absorb exudate.
C. Absorb exudate and allow rapid moisture evaporation.
D. Restrict bacteria from the wound while supporting the tissues.

A

Answer: B
Moisture-retentive occlusive wound dressings such as calcium alginate are recommended for use on exudating wounds (grade III ulcer). They maintain a moist wound environment, absorb exudate, provide autolytic debridement, reduce pain at the wound site, or promote faster healing (reepithelialization).

How well did you know this?
1
Not at all
2
3
4
5
Perfectly
77
Q

77.The PT receives a referral to treat a hospitalized patient with adhesive capsulitis. The patient is recovering from a recent hepatitis B infection. What precautions should the therapist observe?
A. Use of droplet transmission precautions
B. Use contact precautions
C. Ask the patient to wear gloves and avoid contact
D. Wear personal protection equipment (PPE) when transporting the patient to therapy.

A

Answer: B
Hepatitis B is a viral infection that is transmitted by close contact with the infected patient’s body fluids (nasopharyngeal exudate, saliva, sweat, urine, feces, semen, vaginal secretions) and blood and blood products. Health care workers should be vaccinated against the possibility of infection because they are in a high-risk category. Contact precautions should be observed to reduce the risk of microorganism transmission by direct or indirect contact.

How well did you know this?
1
Not at all
2
3
4
5
Perfectly
78
Q

78.A patient with a transtibial amputation is learning to walk using a patellar tendon-bearing
(PTB) prosthesis and is having difficulty maintaining knee stability from heelstrike to foot-flat. Which muscles are MOST likely weak?
A. Hip flexors
B. Back extensors
C. Knee extensions
D. Knee flexors

A

Answer: C
The knee extensors (quadriceps) are maximally active at heelstrike (initial contact) to stabilize the knee and counteract the flexion moment.

How well did you know this?
1
Not at all
2
3
4
5
Perfectly
79
Q

79.A Patient with a history of coronary artery disease and recent myocardial infarction (MI) is exercising in an impatient cardiac rehabilitation program. Because the patient is new, continuous ECG telemetry monitoring is being done. The therapist observes the following. What is the therapist’s BEST course of action?
A. Have the patient sit down, continue monitoring, and notify the physician immediately.
B. Activate the emergency medical response team
C. Have the patient sit down, rest, and then resume the exercise at a lower intensity.
D. Have the patient sit down and send him/her back to the room after a brief rest period.

A

Answer: A
This tracing shows premature ventricular contractions (PVCs) that are multifocal (originating from different irritable ventricular focus). These multiform PVCs pose a potential danger of deteriorating into ventricular tachycardia and ventricular fibrillation (cardiac standstill).
Because the heart is demonstrating a high degree of irritability, the BEST course of action is to stop the exercise, have the patient sit down, continue monitoring carefully, notify medical staff (attending physician) immediately.

How well did you know this?
1
Not at all
2
3
4
5
Perfectly
80
Q

80.A patient recovering from traumatic brain injury (TBI) is unable to bring the right foot up on the step during stair climbing training. What is the BEST choice to promote independent stair climbing for this patient?
A. Practice marching in place
B. Strengthen the hip flexors using an isokinetic training device before attempting stair climbing
C. Passively bring the foot up and place it on the 7-inch step
D. Practice stair climbing inside the parallel bars using a 3-inch step.

A

Answer: D
The most appropriate lead-up activity to promote the skill of stair climbing is practice using a 3-inch step in the parallel bars.

How well did you know this?
1
Not at all
2
3
4
5
Perfectly
81
Q

81.The PT is completing general activity recommendations for a group home of young adults with emotional and behavioral issues. All patients are chemically controlled with either antipsychotic or antidepressant medications. Full-time supervision is available for any activity recommended. Which exercise precautions would be important for the therapist to include?
A. Avoid games with throwing activities to prevent injuries
B. Promote rhythmic movement to soothing music to avoid agitation
C. Promote activities with sequential movements to improve memory
D. Avoid aerobic exercise outdoors when temperature is over 90°F

A

Answer: D
Overheating is detrimental to individuals on antipsychotic or antidepressant medications.

82
Q

82.Following an intial functional examination using the Functional Independence Measure
(FIM), a patient si found to require minimal contact assistance in transferring from sit-to-stand and bed-to-wheelchair. Which of the following accurately documents these results?
A. FIM level 4; completes activity with 75% or more of the effort
B. FIM level 6; completes activity with extra time
C. FIM level 5; completes activity with cueing
D. FIM level 3; completes activity with 50% or more but less than 75% of the effort.

A

Answer: A
This patient’s performance is best categorized by FIM level 4 (completes activity with minimal contact assistance and 75% or more of the effort).

83
Q

83.The therapist is on a home visit, scheduled at lunchtime, visiting an 18-month-old child with moderate developmental delay. The therapist notices that the child and mother are experiencing difficulties with feeding. The child is slumped down in the highchair and is unsuccessfully attempting to use a raking grasp to lift cereal pieces to the mouth. Both the child and the mother are frustrated. Which intervention should the therapist work on FIRST?
A. Recommended that the mother return to breastfeeding for a few more months
B. Work on desensitizing the gag reflex
C. Recommended that the mother feed the child baby food instead of cereal for a few more months
D. Reposition the child in a proper sitting position using postural supports.

A

Answer: D
Feeding can be successful only if the child is positioned in a stable sitting posture: head upright, trunk erect with pelvis neutral and hips flexed to 90°, and feet resting flat. Correct positioning in sitting will facilitate upper extremity function (grasp and release) as well as swallowing.

84
Q

84.A patient is referred to physical therapy with a diagnosis of “frozen shoulder.” Which of the following is the BEST choice of technique to mobilize the shoulder?
A. Lateral glide in neutral position
B. Inferior glide at 55°of abduction
C. Posterior glide at 10° of abduction
D. Inferior glide at 95° of abduction

A

Answer: B
The convex-concave rule for mobilization applies. The MOST effective position to mobilize for improved shoulder abduction is in the resting position (55°). Because the convex humeral head is moving on the concave glenoid, an inferior glide would be MOST appropriate to improve shoulder abduction.

85
Q

85.A patient presents with insidious onset of pain in the jaw that is referred to the head and neck regions. As best as the patient can recall, it may be related to biting into something hard.
Cervical ROM is limited in flexion by 20°, cervical lateral flexion limited to the left by 10°.
Mandibular depression is 10 mm with deviation to the left, protrusion is 4 mm, and lateral deviation is 15 mm to the right and 6 mm to the left. What is the MOST likely diagnosis given this patient’s symptoms?
A. Weak lateral pterygoids on the right
B. Weak lateral pterygoids on the left
C. Capsule-ligamentous pattern of temporomandibular joint (TMJ) on the left.
D. Cervical spine and TMJ capsular restrictions on the left.

A

Answer: C
The capsule-ligamenous pattern of the TMJ is limitation on opening, lateral deviation greater to the uninvolved side and deviation on opening to the involved side. Normal parameters for TMJ measures are 25-35 mm functional and 35-50 mm normal, normal protrusion is 3-6 mm and normal lateral deviation is 10-15 mm.

86
Q

86.A patient has been referred to physical therapy with a diagnosis of cervicogenic headaches.
During the initial examination, the therapist notes clinical findings of a droopy eyelid, constricted pupil (a pinpoint pupil), and lack of sweating on the same side of the face. What is the MOST likely diagnosis based on this group of symptoms?
A. Horner’s syndrome
B. Bell’s palsy
C. Trigeminal neuralgia
D. Myasthenia gravis

A

Answer: A
The hallmark finding of Horner’s syndrome is the clinical triad of ptosis (droopy eyelid), miosis (pupillary contraction), and anhydrosis (lack of sweating), all on the same side of the face and ipsilateral to the lesion. Skin vasodilation is also typically present. Horner’s syndrome results form a lesion that affects the sympathetic pathway to the head. The syndrome may be caused by trauma, interruption of blood supply, tumors, or cluster headaches. Cranial nerve III, the oculomotor nerve, contains parasympathetic fibers that innervate the pupillary constrictor muscle. Therefore, loss of sympathetic innervation of the pupillary dilator muscle results in unopposed pupillary constriction.

87
Q

87.A patient with a recent history of rib fractures suddenly becomes short of breath during a bout of coughing. The patient looks panicked and complains of sharp pain in the left chest. A quick screen shows a deviated trachea to the right, among other signs and symptoms. What is the MOST likely diagnosis based on these symptoms?
A. Pulmonary emboli
B. Pneumothorax
C. Angina
D. Mucous plugging of an airway

A

Answer: B
The deviation of the trachea toward the right with the chest pain on the left is a match of symptoms for the occurrence of a pneumothorax on the left. The history of a rib fracture makes pneumothorax all the more likely.

88
Q

88.A patient presents with supraspinatus tendinitis. After the initial cryotherapy, the therapist decides to apply ultrasound (US). In what position should the therapist place the shoulder joint in order to effectively treat the supraspinatus tendon?
A. Adduction and external rotation
B. Slight abduction and internal rotation
C. Adduction and internal rotation
D. Slight abduction and external rotation

A

Answer: B
Abduction and internal rotation of the shoulder places the supraspinatus tendon in a good position to apply US by exposing the tendon from under the acromion process.

89
Q

89.A patient is referred for postmastectomy rehabilitation. During the initial examination, the therapist observes an irregular area of skin on the patient’s shoulder about 7mm in diameter.
The patient reports that there has always been a mole there but is more prominent lately and that the color has changed, now ranging from black to red to blue. How should the therapist document this finding?
A. Papule
B. Wheal
C. Atypical dysplastic nevus
D. Benign nevus

A

Answer: C
A nevus is a common mole. A changing nevus (atypical dysplastic nevus) that presents with asymmetry (A), irregular borders (B), variation in color (C), diameter > 6 mm (D), and elevation € may be indicative of malignant melanoma (the “ABCDEs” from the American Cancer Society)

90
Q

90.A patient is 2 days, post-left CA and has just been moved from the intensive care unit to a stroke unit. When beginning the examination, the therapist finds the patient’s speech slow and hesitant. The patient is limited to one- and two-word productions, and expressions are awkward and arduous. However, the patient demonstrates good comprehension. What type of speech disorder is this patient exhibiting?
A. Fluent aphasia
B. Global aphasia
C. Nonfluent aphasia
D. Dysarthria

A

Answer: C
This patient is demonstrating classic signs of nonfluent aphasia (broca’s motor or expressive aphasia). It is the result of a lesion involving the third frontal convolution of the left hemisphere. Nonfluent aphasia is characterized by slow and hesitant speech with limited vocabulary and labored articulation. There is relative preservation of auditory comprehension.

91
Q

91.A patient recovering from traumatic brain injury (TBI) is functioning at level IV on the Rancho Los Amigos Levels of Cognitive Functioning Scale (LOCF). During the therapist’s initial examination, the patient becomes agitated and tries to bite the therapist. What is the therapist’s BEST course of action?
A. Postpone the examination for 1 week and then try again
B. Restructure the formal examination so the therapist can complete it in three very short sessions.
C. Document the behaviors and engage in a calming activity.
D. Postpone the examination until later in the day when the patient calms down

A

Answer: C
Patients with TBI in level IV of recovery are confused and agitated. Behavior is bizarre and nonpurposeful relative to the immediate environment. This patient is unable to cooperate directly with formal examination or treatment, lacking both selective attention and memory.
The therapist needs to observe and document the behaviors closely and engage the patient in a calming activity such as slow rocking. A quiet, closed environment is critical.

92
Q

92.A therapist is working on a cardiac care unit in an acute care facility. After exercising a patient recovering from a ventricular infarct, the therapist notices fatigue and dyspnea after mild activity. Later that day, on a return visit, the therapist notices the patient has a persistent spasmodic cough while lying in bed, heart rate is rapid (140), and slight edema is evident in both ankles. The patient appears anxious and agitated. What are these signs and symptoms characteristic of?
A. Developing pericarditis
B. Right ventricular failure
C. Impeding MI
D. Left ventricular failure

A

Answer: D
Typical clinical manifestations of left ventricular failure (CHF) include those described in the case example along with an S3 heart gallop, paroxysmal nocturnal dyspnea, orthopnea, and signs and symptoms of pulmonary edema (marked dyspnea, pallor, cyanosis, diaphoresis, tachypnea, anxiety, and agitation).

93
Q

93.A patient with anterior knee pain has increased adduction and internal rotation at the hip when performing a squat. Which muscle are MOST likely weak, causing this compensatory movement?
A. Knee flexors and extensors
B. Hip adductors and internal rotators
C. Hip and knee flexors
D. Hip abductors and external rotators

A

Answer: D
Decreased strength of the hip abductors and external rotators are common findings in patients with anterior knee pain. Weakness is often demonstrated during a squat with increased hip adduction and internal rotation due to the poor eccentric control of these muscles.

94
Q

94.A patient has been taking corticosteroids (hydrocortisone) for management of adrenocortical insufficiency and is referred to physical therapy for mobility training after a prolonged hospitalization. What are the potential adverse effects from prolonged use of this medication?
A. Hypotension and myopathy
B. Decreased appetite and weight loss
C. Atrophy and osteoporosis
D. Confusion and depression

A

Answer: C
Prolonged use of corticosteroids may result in muscle weakness, osteoporosis, fractures, and joint pain. Large doses are associated with cushingoid changes (e.g., moon face, central obesity, hypertension, myopathy, electrolyte and fluid imbalance). Common central nervous system changes insomnia and nervousness.

95
Q

95.A PT is treating a patient with deep partial-thickness burns over 35% of the body (chest and arms). Wound cultures reveal a bacterial count in excess of 105/g of tissue on the anterior left arm. What are the reasonable expectations for this type of burn wound?
A. With antibiotics, spontaneous healing can be expected.
B. The risk of hypertrophic and keloid scars is low because there is no viable tissue
C. The burn area is pain free because all nerve endings in the dermal tissue were destroyed
D. The infected wound can convert the area to a full-thickness burn.

A

Answer: D
A deep partial-thickness burn will heal in about 3-5 weeks if it does not become infected. An infection typically results in conversion of the wound to a full-thickness burn.

96
Q

96.Recently, a 10-year-old patient has begun walking with supination of the right foot. With the shoe off, the therapist finds a new callus on the lateral side of the metatarsal head of the fifth toe. Which of the following is the BEST choice for orthotic prescription for this patient?
A. Thomas heel
B. Viscoelastic shoe insert with a forefoot medial wedge
C. Viscoeleastic shoe insert with forefoot lateral wedge
D. Scaphoid pad.

A

Answer: C
Supination of the foot (pes cavus) is accompanied by supination of the talocalcaneonavicular
(TCN), subtalar and transversal tarsal joints. It is characterized by an abnormally high arch. The flexible cavus foot generally responds well to orthotic foot control, especially in a child. The best choice is a viscoelastic shoe insert with forefoot lateral wedge.

97
Q

97.A patient is recovering from a right total hip replacement (posterolateral incision, cementless fixation). During initial healing, what is the MOST appropriate type of bed-to-wheelchair transfer to teach this patient?
A. Squat-pivot transfer to the surgical side
B. Lateral slide transfer to the surgical side using a transfer board
C. Stand-pivot transfer to the sound side
D. Stand-pivot transfer to the surgical side.

A

Answer: C
During initial healing, it is important to protect the hip from dislocation or subluxation of the prosthesis. With a posterolateral incision, excessive hip flexion and adduction past neutral are contraindicated. This is minimized by transferring to the sound side.

98
Q

98.During an initial interview and history, a patient with a right CVA seems unconcerned about obvious paralysis of the left arm and leg. When the therapist asks the patient to describe what happened, the patient says “I must have slept wrong and my arm and leg fell asleep.” The patient further tells the therapist, “My family put me in this place so they could go on vacation.” Which type of perceptual disorder BEST characterizes the patient’s responses?
A. Anosognosia
B. Prosopagnosia
C. Spatial relations disorder
D. Somatoagnosia

A

Answer: A
Anosognosia is a perceptual disorder that is characterized by denial, neglect, and lack of awareness of the presence or severity of one’s paralysis.

99
Q

99.The therapist is treating a child with mild developmental delay secondary to 7 weeks prematurity at birth. The child is now 8 months old and is just learning to sit. Which is the BEST choice for a training activity?
A. Sideward protective extension in sitting.
B. Supine tilting reactions
C. Standing tilting reactions
D. Prone tilting reactions

A

Answer: A
Sideward protective extension in sitting is a functional, protective reaction that normally occurs at about the same time as sitting begins.

100
Q

The picture depicts a patient who is learning to take her own pulse. What is the patient doing incorrectly?
A. Taking the pulse with the forearm supinated
B. Palpating the ulnar artery
C. Assessing the pulse with the thumb
D. Palpating the right arm as opposed to the more accurate left arm.

A

Answer: C
The thumb is pulsatile and cannot be used to assess a pulse.

101
Q

A patient with complete spinal cord injury at the level of T11 is on a bowel program.
Which of the following is the MOST effective bowel training program for this patient?
A. Diet and medications to manage a flaccid bowel
B. Digital stimulation of intact defecation reflexes
C. Manual removal of stool from the rectum
D. Medications such as laxative for passive elimination

A

Answer: D
A SCI injury at the level of T11 produces an UMN or spastic bowel with intact spinal defecation reflexes. Bowel and anal sphincters respond to rectal/anal stimulation, enabling a planned bowel elimination program.

102
Q

When performing scoliosis screening in a school setting, what is the optimal age for girls to be screened?
A. 12-14
B. 6-8
C. 9-11
D. 15-17

A

Answer: C
The term adolescent describes children ranging in age from 10-18. The most effective age to screen girls for scoliosis is just before the pubescent growth spurt between 9 and 11 years, when the scoliotic curve can increase dramatically. Boys should be screened between 11 and 13 years of age because of differences in the age of onset of puberty between girls and boys.

103
Q

A patient with a confirmed left C6 nerve root compression due to foraminal encroachment complains of pain in the left thumb and index finger. What is the MOST effective cervical motion to alleviate this patient’s pain?
A. Lower cervical flexion
B. Right rotation
C. Lower cervical extension
D. Left side bending

A

Answer: A
Lower cervical flexion increases the space at the intervertebral foramen, allowing the C6 nerve root to decompress and reduce or alleviate radicular pain.

104
Q

A patient is standing with excessive subtalar pronation. Which of the following indicates the obligatory motions that accompany this condition?
A. Tibial, femoral, and pelvic external rotation
B. Tibial and femoral internal rotation with pelvic external rotation
C. Tibial and femoral external rotation with pelvic internal rotation
D. Tibial, femoral, and pelvic internal rotation

A

Answer: D
With the patient standing, the calcaneus is fixed to the ground. Subtalar joint pronation will occur as the talus plantar flexes, adducts, and inverts. In response to subtalar joint pronation, obligatory internal rotation of the tibia, femur, and pelvis occurs.

105
Q

Which of the following conditions associated with pelvic floor muscle dysfunction may actually worsen when treated with Kegel exercises?
A. Interstitial cystitis
B. Stress incontinence
C. Pelvic organ prolapse
D. Chronic constipation

A

Answer: A
Kegel exercises are prescribed to strengthen weak pelvic floor muscles (levator ani) and to improve the motor control of these muscles. However, in individuals with interstitial cystitis (IC), the pelvic floor dysfunction is usually related to muscles that are too tense or in spasm in response to pain and chronic inflammation of the bladder. This is just the opposite of the too-relaxed state that leads to incontinence. Additionally, there are often tender points or nodules in the pelvic floor muscles of individuals with IC that can be treated with manual techniques.
Therefore the goal of physical therapy in these patients is to relax and lengthen tight pelvic floor muscles and release trigger points.

106
Q

A therapist is applying a symmetrical biphasic pulsed current to the vastus medialis to improve patellar tracking during knee extension. The patient complains that the current is uncomfortable. To make the current more tolerable to the patient, yet maintain a good therapeutic effect, what should the therapist adjust?
A. Current polarity
B. Pulse duration
C. Current intensity
D. Pulse rate

A

Answer: B
Decreasing the pulse duration reduces the electrical charge of each pulse, making the current more comfortable by decreasing the total current applied while maintaining the full therapeutic effect.

107
Q

Which of the following gastrointestinal sources of pain can refer to the shoulder?
A. Esophageal pain
B. Colon or appendix pain
C. Spleen or diaphragmatic pain
D. Gallbladder pain

A

Answer: C
Spleen or diaphragmatic pain can refer to the shoulder.

108
Q

A patient presents with symptoms of uncoordinated eye movements, profound gait and trunk ataxia, and difficulty with postural orientation to vertical. Balance deficits are pronounced in standing with eyes open and eyes closed. Examination of the extremities reveals little change in tone or coordination. What is the likely CNS location of the patient’s dysfunction?
A. Premotor cortex
B. Vestibulocerebellum
C. Spinocerebellum
D. Basal ganglia

A

Answer: B
The symptoms are suggestive of cerebellar dysfunction. The vestibulocerebellum (archicerebellum) is concerned with adjustment of muscle tone in response to vestibular stimuli.
It coordinates muscle actions to maintain postural coordination and balance control along with eye muscle control (all impaired in this example).

109
Q

A patient with a 10-year history of diabetes complains of cramping, pain, and fatigue to the right buttock after walking 400 feet or climbing stairs. When the patient stops exercising, the pain goes away immediately. The skin of the involved leg is cool and pale. The therapist checks the record and finds no mention of this problem. Given this patient’s symptoms, what is the likely diagnosis?
A. Peripheral nerve injury
B. Spinal nerve root impingement
C. Raynaud’s phenomenon
D. Peripheral arterial disease (PAD)

A

Answer: D
Intermittent claudication, often the earliest indication of PAD, is manifested by cramping, pain, or fatigue in the muscles during exercise that is typically relived by rest. The calf muscle is most commonly affected, but discomfort may also occur in the thigh, hip, or buttock. Cessation of pain immediately upon stopping the exercise is characteristic of intermittent claudication, not other spinal problems. With severe disease, however, pain may be present even at rest.

110
Q

A patient with lower back pain has marked elevation of BP and complains of mild to severe mid-abdominal pain that increases upon exertion. Palpation reveals a pulsing mass in the lower abdomen. What is the therapist’s BEST course of action?
A. Discontinue treatment and notify the patient’s physician immediately
B. Provide hot packs to the abdomen to help relive the muscle spasm
C. Instruct the patient to contract his/her physician at the conclusion of therapy
D. Instruct in relaxation exercises because a pulsating mass is not unusual with hypertension

A

Answer: A
This patient is demonstrating signs and symptoms of abdominal aortic aneurysm. Pain is intermittent or constant and can be felt in the mid-abdominal or lower back regions. The pulsating mass is highly significant, and the level of hypertension dramatically increases risk of rupture. This is a serious medical condition; the therapist should notify the physician immediately.

111
Q

A patient presents for the initial examination with an acute and painful shoulder impingement. During the examination, the PT finds significantly increased muscle guarding around the shoulder girdle with difficulty in accurately assessing joint mobility. Which manual therapy technique is the BEST option to use to assist in performing a proper assessment?
A. Maitland grade III posterior glide to the glenohumeral joint
B. Maitland grade II oscillation to the glenohumeral joint
C. Maitland grade III inferior glide to the glenohumeral joint
D. Maitland grade IV inferior glide to the glenohumeral joint

A

Answer: B
Maitland grades I and II are used to improve joint lubrication/nutrition as well as to decrease pain and muscle guarding. A grade II oscillation will promote relaxation, allowing a proper assessment of the patient’s joint mobility.

112
Q

A frail, elderly wheelchair-dependent resident of a community nursing home has a diagnosis of organic brain syndrome, moderate Alzheimer’s type dementia. During the therapist’s interview, the patient demonstrates limited interaction and mild agitation and keeps trying to wheel the chair down the hall. Because it is late in the day, the therapist decides to resume the examination the next morning. How should the therapist document this in the medical record?
A. Disorientation to time and date
B. Inattention as a result of short-term memory loss
C. Frustration because of an inability to communicate
D. Sundowning behaviour

A

Answer: D
A patient with moderate Alzheimer’s type dementia can be expected to exhibit impaired cognition and abstract thinking, sundowning (defined as extreme restlessness, agitation, and wandering that typically occurs in the late afternoon), inability to carry out activities of daily living, impaired judgment, inappropriate social behaviour, lack of insight, repetitive behavior, and a voracious appetite.

113
Q

A patient with a right trans femoral amputation is undergoing prosthetic training. What is the BESt technique to use to improve the patient’s shortened step length on the right?
A. Provide anterior-directed resistance to the right PSIS during swing
B. Provide posterior-directed resistance to the left ASIS during swing
C. Provide posterior-directed resistance to the right ASIS during stance
D. Facilitate the gluteals with tapping over the muscle belly.

A

Answer: C
Light resistance and stretch applied to the pelvis (right ASIS) in a posterior direction during mid-stance to late stance will facilitate forward pelvic rotation on that side and enhance forward movement of the limb during swing.

114
Q

An elderly, frail resident of an extended care facility has intractable constipation. During a scheduled visit from the PT, the patient complains of abdominal pain and tenderness.
Where may this patient experience referred pain?
A. Buttock, thigh, and posterior leg
B. Anterior hip, groin, or thigh region
C. Low back and front of the thigh to the knee
D. Medial thigh and leg

A

Answer: B
Intractable constipation (obstipation) can cause partial or complete bowel impaction, pain, and tenderness in the lower abdomen, referred pain is to the anterior hip groin or thigh region.

115
Q

An adult patient is diagnosed with thoracic outlet syndrome. The patient presents with guarding in the upper trapezius and scalene muscles. Given this situation, which technique would be the MOST effective way to decrease the muscle guarding and provide pain relief?
A. Maitland grade V manipulation of the C6-C7 joint
B. Maitland grade II mobilization of the atlanto-axial joint
C. Maitland grade IlI mobilization of the C6-C7 joint
D. Maitland grade V manipulation of the first rib

A

Answer: D
One needs to be aware that the muscle guarding pattern present with this condition is increased tone of the scalene muscles. These muscles (anterior and middle portions) attach to the first rib. Grade V manipulation is used to decrease muscle guarding and pin in the target area.

116
Q

A patient presents with decreased motion at the occipitoatlantal joint (OA). The PT wants to use the principles of coupled motions that occur in that area of the spine during manual therapy techniques. In order to improve OA mobility, when the occiput is side bent to the right, how should the therapist mobilize C1?
A. Into rotation to the left
B. Into rotation to the right
C. Back into extension
D. Forward into flexion

A

Answer: A
Given the rules of coupled movement in the upper cervical spine, when the occiput is sidebent into one direction, C1 rotates into the opposite direction. Side bending and rotation occur in the same direction from C2-C7 regardless if the spine is in flexion or extension.

117
Q

An apparently healthy individual has several risk factors for coronary artery disease. The client is interested in improving overall fitness and cardiac health. After a graded exercise test, which was asymptomatic, the client is referred for an exercise class. Which is the BEST measure of exercise intensity in a newly tested and exercising individual?
A. Heart rate (HR)
B. Rating of perceived exertion (RE)
C. MET level
D. Respiratory rate

A

Answer: A
A graded exercise test should be performed before commencing an exercise program for all high-risk individuals. The best measurement of exercise intensity in a newly tested and exercising individual is HR.

118
Q

A mother brings her 8-week-old infant to be examined at early intervention clinic because she noticed that the infant was taking steps in supported standing at 2 weeks but is not able to do it now. What should the therapist do given the infant’s symptoms and behaviors?
A. Recommend that the mother bring the infant to a pediatric neurologist
B. Explain that this was abnormal and it is a good sign that it has disappeared.
C. Recommend that a full developmental examination be performed by the early intervention team
D. Explain that this is normal and that this early automatic walking is a newborn response.

A

Answer: D
The mother probably saw the neonatal stepping reflex/automatic walking, which is normal in a newborn but is not exhibited in the older infant probably because of anthropomorphic factors and neural maturation. The age that this response typically disappears in 2-3 months of age.

119
Q

A patient complains of pain with mouth opening that makes it difficult to eat foods that require chewing, Which of the following provides the normal limits of mouth opening that should guide the therapist’s examination?
A. 15-24 mm
B. 35-50 mm
C. 51-65 mm
D. 66-74 mm

A

Answer: B
Average AROM is approximately 35-50 mm. However, only 25-35 mm of opening between the teeth is required for normal everyday activity.

120
Q

The physical therapist is examining the muscle length of the patient’s left hip and knee.
What muscle length test is being shown in the picture below?
A. FABER test
B. Thomas test
C. Noble test
D. Ober test

A

Answer: B
The Thomas test is utilized to test for hip flexor length and to distinguish between one joint and two joint hip flexor tightness. Which low back and sacrum flat on the table, a normal one joint hip flexor length would be with thigh flat on the table. Normal two joint hip flexor length would be 80° of knee flexion.

121
Q

A patient presents with complaints of tingling and paresthesias in the median nerve distribution of the right forearm and hand. The following tests were found negative bilaterally: Adson’s hyperabduction, costoclavicular, Phalen’s, and the ulnar nerve Tinel’s sign. Based on this information, what Is the likely diagnosis?
A. Ulnar nerve entrapment
B. Pronator teres syndrome
C. Thoracic outlet syndrome (TOS)
D. Carpal tunnel syndrome

A

Answer: B
All of these special tests are used to determine neurological compromise of the lower trunk and bracial plexus. Special tests to rule out pronator teres syndrome are (1) passive supination to elongate the pronator, which is thight (this would compress the nerve at that level); and (2) active resistance of pronation, which would compress the enrve as it courses through the pronator muscle belly.

122
Q

A young adult patient is referred to outpatient physical therapy for an insidious onset of thoracic spine stiffness and mild pain. The patient reports a great deal of difficulty moving the mid-back region in the morning and states that thus has become progressively worse over the past 6 months. The neurological exam is negative. Based on these symptoms, what is the likely diagnosis?
A. Pneumothorax
B. Thoracic compression fracture
C. Ankylosing spondylitis
D. Lyme disease

A

Answer: C
Ankylosing spondylitits is a rheumatic disease of the spine (spondyloarthropathy). Primary symptoms include back pain and back stiffness, especially in the morning. Chronic inflammation and irritation of the spinal joint results in stiffening of the joints and eventually abnormal fusion (ankylosis).

123
Q

A young patient presents with primary lymphedema of the right lower extremity. What is the BEST choice for initial exercise?
A. Treadmill walking
B. Treadmill jogging
C. Exercising on a stair climbing machine
D. Step aerobics

A

Answer: A
Initially, the patient should begin with beneficial low-risk activities (e.g., lymphedema exercise, walking, easy biking, swimming, water aerobics, or Tai Chi). Exercise should always be performed with a compression garment or compression bandages.

124
Q

An elderly patient is recovering from a right CA and demonstrates strong spasticity in the left upper extremity. The therapist wants to reduce the expected negative effects of spasticity in the left upper extremity while the patient is working on sitting control. What is the BEST position for the upper extremity?
A. Left elbow flexed with arm resting on supporting pillow, positioned on the patient’s lap
B. Affected upper extremity extended and internally rotated with the hand at the side
C. Left shoulder abducted and externally rotated with elbow extended and weight supported on the palm of the hand
D. Left shoulder adducted and internal rotation with arm extended and hand resting on the thigh

A

Answer: C
In the upper extremity, spasticity is typically strong in scapular retractors, shoulder adductors, depressors, and internal rotators; elbow flexors and forearm pronators; and wrist and finger flexors. The patient should be positioned opposite the expected pattern.

125
Q

During an initial examination, the therapist occludes vision by having the patient close the eyes. What can the therapist effectively examine?
A. Discriminative touch and fast pain but not proprioception
B. Vestibular/visual/somatosensory integration
C. Conscious proprioception but not discriminative touch
D. Somatosensory integrity

A

Answer: D
The term “somatosensation” refers to conscious relay pathways for discriminative touch, conscious proprioception, fast pain, and discriminative temperature. Sensory examination must rule out vision in order to establish the reliability of testing sensory integrity of these pathways.

126
Q

An adolescent felt a “clunk” in the lumbar spine 2 weeks ago while lifting weights. There was immediate right lumbar pain and spasm. Posteroanterior 1 mm anterior to L5 on the lateral views. Which of the following imaging techniques would give the PT the best information regarding a diagnosis and formulating a plan of care for this individual?
A. Posteroanterior computed tomography (CT) scan
B. Posteroanterior T1 magnetic resonance imaging (MRI)
C. Bilateral oblique radiographs
D. Right oblique radiograph

A

Answer: C
The clinician should suspect a spondylolisthesis. A spondylolisthesis is a forward slippage of a vertebra due to a bilateral defect in the pars interarticularis. Causes include congenital, acute fracture, or degenerative conditions. The degree of forward slippage is graded on a 1-4 scale (4 being the most severe or a 75%-100% slippage) from the lateral view. It is unclear from the description whether there is an actual spondylolisthesis. Bilateral oblique views with a radiograph are needed to see whether there is a fracture at the pars interarticularis bilaterally.
This is known as the “Scottie dog” defect.

127
Q

A 28-year-old professional mountain bike rider presents to cardiac rehabilitation after suffering a myocardial infarction 8 weeks ago. Which graded exercise test would be MOST beneficial in order to assess aerobic capacity in this patient in order to create an aerobic conditioning program?
A. Bruce protocol
B. Step test
C. Lower extremity ergometry
D. 6-minute walk test

A

Answer: C
The patient is well-trained on a mountain bike and will benefit from LE ergometry testing and a bike training program to facilitate return to his role.

128
Q

A patient with trigeminal nerve neuralgia (CN V) is referred to the physical therapist.
What are the expected examination findings?
A. Sudden severe pains in the ophthalmic division of CN V
B. Paroxysmal and severe pain originating from the mandibular or maxillary divisions of
CN V
C. Unilateral sensory loss of the ophthalmic division of CN V
D. Bilateral sensory loss of CN V in all three divisions

A

Answer: B
Trigeminal neuralgia (tic douloureux) is a condition characterized by sudden, severe pain occurring in the distribution of the trigeminal nerve (CN V). It typically occurs in the maxillary or mandibular division on one side of the face.

129
Q

A patient with hypothyroidism and poor drug compliance is referred to physical therapy following a fall. What symptoms might be evident during exercise based on this diagnosis?
A. Paresthesias of the lower limbs
B. Elevated cardiac output
C. Sinus tachycardia and arrhythmias
D. Myalgia and weakness

A

Answer: D
Hypothyroidism results in a decreased metabolic rate and is likely to produce exercised-induced myalgia and weakness (rhabdomyolysis).

130
Q

A group of researchers utilized meta-analysis to identify the evidence for aerobic fitness exercises in the management of fibromyalgia. Thirteen randomized, controlled trials (RCTs) and three controlled clinical trials (cohort studies and case control studies) were selected.
What is the main difference between the two types of trials?
A. Duration of the studies
B. Use of multiple centers versus single center trials
C. Length of the studies
D. Use of randomization of subjects.

A

Answer: D
The main difference between the two types of trails is randomization of subjects into experimental and control groups (RCT).

131
Q

A patient with a complete spinal cord injury at the level of TI (ASIA A) is in the community phase of mobility training. In order for the patient to navigate a 4-inch-height curb with the wheelchair, what should the therapist tell the patient to do?
A. Ascend backward with the large wheels first
B. Descend backward with the trunk upright and arms hooked around the push handles
C. Lift the front casters and ascend in a wheelie position
D. Place the front casters down first during descent

A

Answer: C
Curbs are ascended in the wheelie position (front casters lifted with push forward on rear wheels). Individuals must learn to use momentum and a strong push to elevate the front casters of the chair and propel the wheelchair up the curb.

132
Q

A patient with a long history of systemic steroid use for asthma control is hospitalized with pneumonia. Which of the following is a contraindication to percussion?
A. Barrel chest
B. BP > 140/90
C. Intercostal muscle wasting
D. Decreased bone density

A

Answer: D
The only one that is a contraindication to percussion would be decreased bone density, because a rib fracture might be a possible result

133
Q

The physical therapist is examining the patient’s left ankle. The test pictured below is testing the integrity of which structure?
A. Anterior tibiofibular ligament
B. Tibiotalar ligament
C. Calcaneofibular ligament
D. Anterior talofibular ligament

A

Answer: D
The special test being performed is the anterior drawer test of the ankle. This test is designed primarily to test for injuries to the anterior talofibular ligament. This is the most frequently injured ligament in the ankle. When the foot is in slight plantarflexion position the anterior talofibular ligament is perpendicular to the long axis of the tibia. A positive anterior drawer test may be obtained with a tear of only the anterior talofibular ligament, but anterior translation is greater if the calcaneofibular ligament is also torn. Ideally, the knee should be placed in 90° of flexion to alleviate tension on the Achilles tendon

134
Q
  1. A patient with possible ligamentous injury of the knee presents with excessive tibial external rotation. Which ligament is MOST LIKELY to be injured?
    A. Posterior cruciate
    B. Medial patello-femoral
    C. Anterior cruciate
    D. Medial collateral
A

Answer: D
The medial collateral ligament prevents external rotation and provides stability to the knee.
Excessive external rotation (ER) would represent injury to either the medial collateral ligament
(MCL) or the lateral collateral ligament (LCL).

135
Q

A patient suffered a meniscus injury and underwent recent meniscal repair surgery. What is the earliest the patient will be allowed to weight bear after surgery?
A. At the end of postsurgical week 1
B. Immediately
C. At the end of postsurgical week 3
D. At the end of postsurgical week 2

A

Answer: C
After a meniscal repair, patients should be non-weight bearing for 3-6 weeks.

136
Q
  1. A therapist is treating a patient with a diagnosis of right shoulder rotator cuff tendinitis.
    The findings of a work site ergonomic assessment indicate that the worker is required to perform repetitive reaching activities above shoulder height. Which of the following is the MOST beneficial work site modification?
    A. Reposition the height of the shelf and items to below shoulder height
    B. Provide the worker with a taller, sit-stand chair
    C. Allow the worker to take more frequent rests to avoid overuse
    D. Provide the worker with a standing desk for daily activities
A

Answer: A
Work stations should be designed to accommodate the persons who actually work on the job.
Work stations should be easily adjustable and designed to be comfortable for the worker. In this case, lowering the height of the shelf for frequent use is best.

137
Q

The patient has a fifth rib that is “stuck” in the position of maximal inspiration. Which technique is BEST to improve the rib mobility and assist it in returning to its resting position?
A. Maitland grade IV mobilization of the head of the rib at the costovertebral joint in the superior direction
B. Maitland grade II mobilization of the head of the rib at the costovertebral joint in the superior direction
C. Maitland grade IV mobilization of the head of the rib at the costovertebral joint in the inferior direction
D. Maitland grade II mobilization of the head of the rib at the costovertebral joint in the inferior direction

A

Answer: A
With inspiration, the lateral portion of the ribs moves up and the head moves down; to bring it back to a neutral position, the head needs to glide superiorly, allowing the lateral part of the rib to lower with expiration. Grade IV mobilizations are used to improve joint mobility.

138
Q

A patient with a fibular fracture complains of weakness in the ® LE following cash removal. Examination reveals measurable loss of muscle bulk (2-inch girth difference between the right and left legs). The therapist suspects neurogenic atrophy and next examines tone. Which finding is consistent with this diagnosis?
A. Normal tone
B. Hypotonia
C. Dystonia
D. Hypertonia

A

Answer: B
Hypotonia is one of a number of signs and symptoms of lower motor neuron (LMN) injury that also include neurogenic atrophy and weakness (as in this case), along with decreased or absent reflexes and fasciculations.

139
Q

A patient has a 10-year history of peripheral vascular disease (PVD) affecting the right lower extremity. During auscultation of the popliteal artery, what would the therapist expect to find?
A. A positive Homan’s sign
B. Intense pain and cramping
C. A bruit
D. 4+ pulses

A

Answer: C
A bruit is a swishing sound that occurs in the presence of narrowing of an artery. It is a characteristic finding of PVD present on auscultation.

140
Q

When performing the Thomas test, the patient’s thigh does not touch the table, indicating limited hip extension. The amount of limited hip extension does not change when the ipsilateral knee is extended. What is the range-limiting muscle?
A. Rectus femoris
B. Tensor fascia lata
C. Biceps femoris
D. iliopsoas

A

Answer: D
The iopsoas is a one joint hip flexor muscle and will limit hip extension regardless of the amount of knee flexion.

141
Q

A teenager presents to the clinic with vague left hip and groin pain that worsens with weight bearing. The PT’s examination reveals limited and painful hip internal rotation, antalgic gait, and a weak gluteus medius. Based upon this clinical presensation, what is the MOST likely diagnosis?
A. Gluteus medius muscle strain
B. Oligoarticular juvenile rheumatoid arthritis (JRA)
C. Slipped capital femoral epiphysis (SCFE)
D. Legg-Calve-Perthes-disease

A

Answer: C
The SCFE age range is 10-16 years of age, and the male to female ratio is 3:1. The incidence of left hip to right hip is 2:1, with 30% bilateral. The best examination findings include pain that worsens with weight bearing, limited and painful hip internal rotation, and weak hip abductors.

142
Q

A patient with a transfemoral amputation and an above-knee prosthesis demonstrates knee instability while standing. The patient’s knee buckles easily when performing weight shifts. What is the MOST likely cause of the problem?
A. Weak gluteus medius
B. Prosthetic knee set too far anterior to the TKA line
C. Tight extension aid
D. Prosthetic knee set too far posterior to the trochanter-knee-ankle (TKA) line

A

Answer: B
In order to increase stability of the knee, the prosthetic knee is normally aligned posterior to a line extending from the trochanter to the ankle (TKA line) a knee set anterior to the TKA line will buckle easily.

143
Q

An older adult at risk for falls has undergone a structured home-based exercise program that consisted of standing balance training and strengthening exercises. Which measure is the BEST choice to document improvements?
A. Timed Up & Go Test
B. Berg Balance Test
C. 6-Minute Walk Test
D. Performance-Oriented Mobility Assessment (Tinetti).

A

Answer: B
The Berg Balance Test is a 14-item test of static and dynamic balance in sitting and standing. It also examines sit-to-stand and stand-to-sit transitions. It does not include items examining gait.
Gait was not part of this person’s training program.

144
Q

A middle-aged patient complains of “throbbing pain” in the lumbar region with activities upon exertion, such as walking up a flight of steps or playing tennis. The patient expresses no complains of pain with bending, twisting, sitting, standing, or walking any distance.
Active movements of the lumbar spine are full and pain free. Provocation testing is negative.
Neurological signs are unremarkable. There is no significant tenderness to palpation. What is the MOST likely diagnosis for this patient?
A. Lumbar disc herniation
B. Aortic aneurysm
C. Quadratus lumborum muscle strain
D. Sacroiliac joint sprain.

A

Answer: B
Throbbing low back pain that occurs only with activities that increase the heart rate is a red flag for aortic aneurysm. Medical referral is indicated.

145
Q

A patient who is undergoing spinal cord rehabilitation is viewed as uncooperative by staff. The patient refuses to complete the training activities out lined to promote independent functional mobility. A review of history reveals that previously the patient was the director of a company with a staff of 20. What is the MOST appropriate action the therapist can take?
A. Have the patient work with a supervisor who is a person in authority
B. Refer the patient to a support group before resuming rehabilitation
C. Involve the patient in goal setting and structuring the training session
D. Carefully structure the activities and slow down the pace of training

A

Answer: C
An andragogical approach is best. The patient is an adult learner who should be allowed to share in the responsibility for planning the learning experience and goal setting. The therapist should held clarify the problem, structure the learning environment, and provide necessary
resources.

146
Q

A group of 10 patients is recruited into a study investigating the effects of relaxation training on blood pressure (BP). One group of patients is scheduled to participate in a supervised cardiac rehabilitation program that includes relaxation training three times a week for 12 weeks. The other group of patients is instructed to perform activities as usual.
At the conclusion of the study, there was no significant difference between the groups; BP decreased significantly in both groups. What is the MOST accurate interpretation of this study?
A. The rehabilitation group was not properly monitored
B. Both groups had BPs initially so high that reductions should have been expected
C. The activities of the nonrehabilitation group were not properly monitored and may account for these results.
D. Cardiac rehabilitation is not effective in reducing BP

A

Answer: C
To ensure adequate control, the researcher should attempt to remove the influence of any variable other than the independent variable in order to evaluate its effect on the dependent variable. In this study, the investigator did not adequately investigate the usual activities of the control group. The small number of subjects may also have contributed to lack of significance.

147
Q

A therapist is examining a patient newly diagnosed with multiple sclerosis. The patient experiences an intense shock-like pain throughout the body when the neck is passively flexed. What is the correct term for the therapist to use in order to document these findings?
A. Lhermitte’s sign
B. Dysesthesias
C. Paresthesias
D. Uthoff’s sign

A

Answer: A
Lhermitte’s sign is a sign of posterior column damage in the spinal cord; flexion of the neck produces an electric shock-like painful sensation running down the spine and into the LEs. It can be seen in patients with MS.

148
Q

Which cluster of examination findings would indicate that a patient is in decompensated heart failure?
A. S4 heart sound, wheezes on lung auscultation, decreased jugular vein distension
B. S4 heart sound, crackles on lung auscultation, increased jugular vein distension
C. S2 Heart sound, crackles on lung auscultation, decreased jugular vein distension
D. S2 heart sound, wheezes on lung auscultation, increased jugular vein distension

A

Answer: B
S4 indicates a pathology in the ventricle, as is present in a patient with heart failure. Crackles on auscultation are often heard in a patient with heart failure because they have compressive atelectasis due to pulmonary edema. When cued to take a deep breath, they can create enough pressure to open the alveoli, which causes the crackling sound. It is possible that a patient in heart failure will also have wheezes if there is significant pulmonary edema causing the alveoli to fill with fluid. With decompensated heart failure, there will be a back-up of fluid throughout the cardiac and pulmonary system, which can extend into the peripheral circulatory system.
This will cause increased distension of the jugular veins.

149
Q

A physical therapist is observing a child who is typically developing and has just begun to walk within the last month. Which of the following is expected in a child just learning to walk?
A. Neutral hip position
B. Bilateral hip adduction
C. Genu valgum
D. External rotation of the hips

A

Answer: D
Children who have just begun to walk externally rotate their hips (toe out) for balance. This increases the base of support.

150
Q

Where should a therapist’s hand/fingers be located for posteroanterior mobilization to improve down-gliding/closure of the T7-8 facet joints?
A. Spinous process of T6
B. Transverse processes of T8
C. Spinous process of T8
D. Transverse processes of T7

A

Answer: B
The axis of motion for mid-thoracic vertebrae is above the spinous processes and below the transverse processes. Therefore, if down-gliding/closure of T7-8 vertebral segment is required, the therapist’s hand placement should be at the transverse process of T8 or the spinous process of T7

151
Q

A patient complains of waking up several times at night from severe “pins and needles” in the right hand. On awakening, the hand feels numb for half an hour and fine hand movements are impaired. The therapist’s examination reveals sensory loss and paresthesias in the thumb, index, middle, and lateral half of the ring finger, and reduced grip and pinch strength. Some thenar atrophy is present. Based on these examination findings, what is the MOST appropriate diagnosis?
A. Pronator teres syndrome
B. Ulnar nerve entrapment
C. Thoracic outlet syndrome
D. Carpal tunnel syndrome

A

Answer: D
Carpal tunnel syndrome is the result of compression of the median nerve under the flexor retinaculum at the wrist. It is characterized by thenar atrophy and sensory loss. Symptoms are worse at night and include burning, tingling, pins and needles, and numbness into the median nerve sensory distribution (palmar and dorsal thumb, index, middle, and lateral half of the ring finger).

152
Q

A patient is referred for physical therapy with a diagnosis of degenerative joint disease
(DJD) affecting C2 and C3. The patient complains of pain and stiffness in the cervical region and transient dizziness with some cervical motions. What is the BEST initial examination procedure?
A. Vertebral artery test
B. Adson’s maneuver
C. Lhermitte’s test
D. Oppenheim’s test

A

Answer: A
The vertebral artery test checks the integrity of the blood flow through the artery in the cervical region. Because the patient is experiencing symptoms of circulatory disturbance and a unilateral pull could compress the left cervical structures, the vertebral artery test is an appropriate screening test. The test consist of passively placing the patient’s head in extension and side flexion in a supine position. Then the head and neck are slowly rotated to the laterally flexed side and held for 30 seconds. Some of the positive signs may be syncope, lightheadedness, nystagmus, or visual disturbances. Though there are some doubts about the sensitivity/specificity of this test, the patient’s initial complaint of dizziness with some cervical movements would indicate that it be applied in this case.

153
Q

A young, otherwise healthy, adult is recovering from a complete spinal cord injury (ASIA A) at the level of L4. What are the functional expectations for this individual?
A. Ambulation using bilateral AFOs and canse
B. Ambulation using bilateral KAFOs, crutches, and a swing-through gait
C. Ambulation using reciprocating gait orthoses and a reciprocating walker
D. Ambulation using bilateral KAFOs and a reciprocating walker

A

Answer: A
A spinal cord lesion at the level of L4 is considered an LMN injury (cauda equine injury). Intact movement include hip flexion, hip adduction, and knee extension. The quadriceps become FULLY innervated at L4. This patient can be expected to be a functional ambulatory using bilateral AFOs and crutches or canes.

154
Q

To examine a patient with a suspected deficit in graphesthesia, what should the therapist ask the patient to identify with eyes closed?
A. Different objects placed in the hand and manipulated
B. The vibrations of a tunning fork when placed on a bony prominence
C. A series of letters traced on the hand
D. Differently weighted, identically shaped cylinders placed in the hand

A

Answer: C
Graphesthesia is the ability to recognize numbers, letters, or symbols traced on the skin.

155
Q

A patient recovering from stroke is ambulatory without an assistive device and demonstrate a consistent problem with an elevated and retracted pelvis on the affected side.
Which manual therapeutic exercise procedure is the BEST choice to remediate this problem?
A. Provide downward compression during stance
B. Utilize light resistance to posterior pelvic elevation during swing
C. Provide anterior-directed pressure during swing
D. Utilize light resistance to forward pelvic rotation during swing

A

Answer: D
An elevated and retracted pelvis is a common problem during gait for many patients recovering from a stroke. Providing light resistance to forward pelvic rotation actively engages those muscles and reciprocally inhibits the spastic retractors.

156
Q

A therapist suspects lower brain stem involvement in a patient with amyotrophic lateral sclerosis (ALS). Examination findings reveal motor impairments of the tongue with ipsilateral wasting and deviation on protrusion. These findings confirm involvement of which cranial nerve?
A. Hypoglossal
B. Glossopharyngeal
C. Vagus
D. Spinal accessory

A

Answer: A
The hypoglossal (CN XII) controls the movements of the tongue. Ipsilateral wasting and the deviation to the ipsilateral side on protrusion are indicative of damage.

157
Q

A baseball pitcher report insidious onset of symptoms characteristic of impingement, including catching and popping in the throwing arm. Examination reveals that glenohumeral passive internal rotation is painful and limited to 30°. External rotation is less symptomatic and has 130° of passive range. What is the BEST initial course of action for the therapist?
A. Recommend an MRI
B. Mobilize the glenohumeral joint to increase internal rotation ROM
C. Begin elastic resistance exercises for impingement
D. Recommend an anteroposterior radiograph

A

Answer: A
The therapist should suspect a labral tear. This is a common finding among pitchers and athletes who do a lot of throwing (especially those who present with abnormal ROM findings and instability symptoms of popping and catching). Although this athlete may present with impingement, an MRI is warranted to fully diagnose the condition and to develop an appropriate treatment plan.

158
Q

A woman is hospitalized in the intensive care unit with multiple closed and open fractures after a motor vehicle accident. A review of her medical record reveals the following laboratory values: haematocrit 28%, haemoglobin 10g/ 100 mL and serum white blood cell (WBC) count 12,000/mm°. What is the MOST accurate interpretation of these findings?
A. Only serum WBC is abnormal
B. Only haematocrit values are abnormal
C. Hematocrit and haemoglobin values are abnormal; WBC is normal
D. All values are abnormal

A

Answer: D
The haematocrit value is abnormally low; women average 42% with a normal range from 37-47%. The haemoglobin value is also abnormally low; women average 12-16g/100 mL of blood.
The low values are most likely due to blood loss. The serum WBC (leukocytes) is abnormally high; normal values are 5,000 to 10,000/mm’. The elevated count (>10,000) indicates acute infection

159
Q

A patient with a transfemoral amputation is unable to wear a total contact prosthesis for the past 4 days. Examination of the residual limb reveals erythema and edema extending over most of the lower anterior limb. The patient tells the therapist that the limb is very itchy and painful. What is the MOST likely cause of these symptoms?
A. Impetigo
B. Herpes Zoster
C. Cellulitis
D. Dermatitis

A

Answer: D
This patient is exhibiting symptoms of contact dermatitis. Primary treatment is removal of the offending agent (in this case, the total contact prosthesis) and treatment of the involved skin with lubricants, topical anesthetics, and/or steroids. The patient may require a thin sock if the problem does not resolve.

160
Q

A PT is treating a patient who lacks wrist extension. The cause of the impingement is a problem at the radiocarpal joint with a lack of arthrokinematic motion necessary for proper wrist extension. What direction should the proximal aspect of the scaphoid/lunate glide?
A. In a dorsal direction relative to the radius
B. Radially
C. Ulnarly
D. In a palmar direction relative to the radius

A

Answer: D
The correct description of the arthrokinematic motion occurring at the radiocarpal joint lacking in this patient is the palmar glide. Due to the anatomical structure of the bones forming this joint, the convex scaphoid and lunate articulate with the concave radius, resulting in opposite roll and glide during extension.

161
Q

Following a hip fracture that is now healed, a patient resents with weak hip flexors (2/5). All other muscles are within functional limits. Based on these findings, what should the therapist expect the patient may display during gait?
A. Backward trunk lean
B. Forward trunk lean
C. A circumducted gait
D. Excessive hip flexion

A

Answer: C
Circumduction is a compensation for weak hip flexors or an inability to shorten the leg (weak knee flexors and ankle dorsiflexors). Hip hiking can also compensate for an abnormally long leg (lack of knee flexion and dorsiflexion).

162
Q

An elderly patient with a left trans femoral amputation complains that when sitting, the left foot feels cramped and twisted. What is the therapist’s BEST choice of intervention?
A. Appropriate bed positioning with the residual limb in extension
B. Iontophoresis to the distal residual limb using hyaluronidase
C. Hot packs and continuous US to the residual limb
D. Icing and massage to the residual limb

A

Answer: D
This patient is experiencing phantom pain, a common occurrence seen in as many as 70% of patients with lower limb amputations. Treatment interventions can include icing, pulsed US, TENS, or massage. Medical interventions for prolonged and intractable pain can include injections and surgical procedures (rhizotomy, neurectomy).

163
Q

A researcher uses a group of volunteers (healthy college students) to study the effects of therapy ball exercises on ankle ROM and balance scores. Twenty volunteers participated in the 20-minute ball exercise class three times a week for 6 weeks. Measurements were taken at the beginning and end of the sessions. Significant differences were found in both sets of scores and reported at the local physical therapy meeting. What is the MOST accurate interpretation of this study and its results?
A. The reliability of the study was threatened with the introduction of systemic error of
measurement
B. The Hawthorne effect may have influenced the outcomes of the study
C. The validity of the study was threatened with the introduction of sampling bias
D. Therapy ball exercises are an effective intervention to improve ankle stability after chronic ankle sprain

A

Answer: C
The investigator used a sample of convenience and therefore introduced systematic sampling error (a threat to validity. Random selection of subjects would improve the validity of this study.

164
Q
  1. A patient has a very large right-sided bacterial pneumonia. Oxygen level is dangerously low. Which of the following body positions is the MOST likely to improve this patient’s arterial oxygen pressure (Pa02)?
    A. Prone-lying with the head of the bed in the trendelenburg position
    B. Right side-lying with the head of the bed in the flat position
    C. Left side-lying with the head of the bed in the flat position
    D. Supine-lying with head of the bed in the trendelenburg position
A

Answer: C
In order to match perfusion and ventilation, the therapist needs to place the unaffected side in a gravity-dependent position or that of left side-lying.

165
Q

An examination of a patient reveals drooping of the shoulder, rotary winging of the scapula, an inability to shrug the shoulder, and complaints of aching in the shoulder. Based on these findings, what is the MOST likely cause of these symptoms?
A. Strains of the serratus anterior
B. A lesion of the spinal accessory nerve
C. A lesion of the long thoracic nerve
D. Muscle imbalance

A

Answer: B
Rotary winging occurs when the inferior angle of one scapula is rotated farther from the spine than the inferior angle of the other scapula. The shoulder drooping and inability to shrug the shoulder are secondary to a lesion of the spinal accessory nerve (CN XI), which innervates the trapezius muscle.

166
Q

A patient with spastic hemiplegia is referred to the therapist for ambulation training. The patient is having difficulty with standing up from a seated position as the result of co-contraction of the quadriceps and hamstrings during the knee and hip extension phase. The therapist wishes to use biofeedback beginning with simple knee extension exercise in the seated position. The plan is to progress to sit-to-stand training. What is the proper initial biofeedback protocol?
A. Low-detection sensitivity with recording electrodes placed far apart
B. High-detection sensitivity with recording electrodes placed closely together
C. Low-detection sensitivity with recording electrodes placed closely together
D. High-detection sensitivity with recording electrodes placed far apart

A

Answer: C
By initially placing the electrodes close together, the therapist decreases the likelihood of detecting undesired motor units from adjacent active muscles (crosstalk). By setting the biofeedback sensitivity (gain) low, the therapist would decrease the amplitude of the signals generated by the hypertonic muscles and keep the EMG output from exceeding a visual and/or auditory range (scale)

167
Q

Examination of a patient recovering from stroke reveals a loss of pain and temperature sensation on the left side of the face along with loss of pain and temperature sensation on the right side of the body. All other sensations are normal. What is the likely location of the lesion?
A. Right cerebral cortex or internal capsule
B. Midbrain
C. Left cerebral cortex or internal capsule
D. Left posterolateral medulla

A

Answer: D
A lesion in the posterolateral medulla causes mixed sensory loss (described in this case). Pain and temperature are affected, whereas discriminative touch and proprioception are not (the medial lemniscus is not involved).

168
Q
  1. A patient is taking the drug baclofen to control spasticity after a complete spinal cord injury at T10. This medication can be expected to decrease muscle tone and pain. What are the possible adverse effects of taking baclofen?
    A. Urinary retention and discomfort
    B. Drowsiness and muscle weakness
    C. Hypertension and palpitations
    D. Headache with visual auras
A

Answer: B
Baclofen, used in the management of spasticity, can produce CNS depression (drowsiness, fatigue, weakness, confusion, vertigo, dizziness, and insomnia), occurring in less than 10% of patients. Additional adverse effects can include hypotension and palpitations and urinary frequency. Vomiting, seizures, and coma are signs of overdosage.

169
Q

A teen-aged female distance runner presents a history of stress fractures and general leg pain. Her parents think it may be due to overtraining. Based on this subjective information, what should the physical therapist question the patient about next?
A. Recent growth spurts
B. Menses and eating habits
C. Type of running shoe
D. Participation in other sports

A

Answer: B
The female athlete triad is a syndrome common in females 14-20 years of age that consists of three related conditions: disordered eating habits, irregular or absent menstrual period, and osteopenia (thinning of the bone density). It is imperative that the therapist inquire about menses and eating habits as they are two of the three hallmark signs of the female athlete triad.
It would also be helpful for the physical therapist to inquire more about muscle pain, medications, and details if training regimen.

170
Q

A patient is exercising in a phase 3 outpatient cardiac rehabilitation program that utilizes circuit training. One of the stations utilizes weights. The patient lifts a 5-Ib weight, holds it for 20 seconds, and then lowers it slowly. The therapist correct the activity and tells the patient to reduce the length of the static hold. Which of the following BEST describes the expected effects of isometric exercise?
A. Reduced normal venous return to the heart and elevated BP
B. Abnormal oxygen uptake
C. Lower heart rate (HR) and arterial blood pressure (BP)
D. Higher HR and arterial BP

A

Answer: D
Dynamic exercise facilitates circulation, while isometric (static) exercise hinders blood flow, producing higher HRs and arterial BPs. Rise in BP is related to degree of intensity (effort).

171
Q

A patient complains of pain in the right lower aspect of the lateral rib cage. The chief complaint is intense pain, which also occurs at night. The pain lasts for approximately 15 minutes and then subsides. There was no mechanism of injury and no pain with activity during the day. Active and passive motion of the thoracic spine and costal cage is normal and pain free. Mild tenderness is present inferior to the right lateral ribcage. What is the MOST likely diagnosis for this patient?
A. Intercostal muscle strain
B. Costochondritis
C. Systemic disease
D. Abdominal muscle strain

A

Answer: C
This history is an insidious onset, with no injury and no pain with activity. There is no reproduction of symptoms with the physical exam. This is typically a red flag for a systemic condition that should be referred to a medical doctor.

172
Q

An elderly male patient recovering from a fractured hip repaired with ORIF has recently been discharged home. During a home visit, his wife tells the therapist that he woke up yesterday morning and told her he couldn’t remember much. Upon examination, the therapist finds some mild motor loss in his right hand and anomia. The therapist affirms the presence of short-term memory loss. What is the therapist’s BEST course of action?
A. Advise the family to document and record any new problems that they notice over the next week and then report back to the therapist.
B. Ignore the findings because they are expected after surgical anesthesia
C. Refer him to his physician immediately because the therapist suspects a stroke.
D. Refer him to physician because the therapist suspects Alzheimer’s type dementia

A

Answer: C
The presence of focal signs (incoordination, anomia) with cognitive signs (memory loss) is indicative of impaired brain function and may be the result of small strokes. This is the most likely choice given the spotty symptoms he presents with as well as their sudden onset.

173
Q

An adolescent with a 4-year history of type 2 diabetes is insulin dependent and wants to participate in cross-country running. The PT working with the school team advises the athlete to measure plasma glucose concentrations before and after running. What additional advice should the therapist give this student athlete?
A. Increase insulin dosage immediately before running
B. Consume a carbohydrate before or during practice to avoid hypoglycaemia
C. Avoid carbohydrate-rich snacks within 12 hours of a race
D. Consume a carbohydrate after practice to avoid hyperglycemia

A

Answer: B
During exercise of increasing intensity and duration, plasma concentrations of insulin progressively decrease. Exercise-induced hypoglycemia (abnormally low levels of glucose in the blood) is common for exercising athletes with diabetes. Hypoglycemia associated with exercise can occur up to 48 hours after exercise. To counteract these effects, the individual may need to reduce his insulin dosage or increase carbohydrate intake before or after running.
Consuming a carbohydrate product before or during the race will have a preventive modulation effect on hypoglycaemia.

174
Q

A male patient is referred to outpatient physical therapy for lower back pain. During the patient interview, he describes a recent increased difficulty with urinating that does not affect his lower back pain symptoms. Neurological screening examination is normal and Murphy’s sign is negative. Based on this clinical scenario, what may this patient’s low back pain be associated with?
A. Lower urinary tract
B. First lumbar nerve root
C. Kidney
D. Sacro-iliac joint

A

Answer: A
The lower urinary tract can refer symptoms to the lumbar spine region, and the recent report of increased difficulty with urination is a key factor.

175
Q
  1. A PTA is ambulating a patient with a stroke using a walker. The patient is unsteady and fearful of falling. The patient does not appear to understand the correct gait sequence. What is the supervising PT’s BEST course of action?
    A. Intervene and teach the correct sequence because the PTA is apparently unable to deal with this special situation
    B. Instruct the PTA to have the patient sit down and utilize mental practice of the task
    C. Tell the PTA and patient to stop the ambulation and work on dynamic balance activities instead
    D. Instruct the PTA to use a distributed practice schedule to ensure patient success.
A

Answer: B
Mental rehearsal (mental practice) is the best strategy to have the patient learn the correct sequence. In the non-weight-bearing position, the patient’s anxiety is lessened, leaving the patient free to concentrate on the task at hand.

176
Q

A therapist is reviewing x-rays from a patient with a trimalleolar fracture. What are the BEST radiographic views to visualize this bony fracture?
A. Oblique and lateral
B. Anteroposterior and lateral
C. Posteroanterior and lateral
D. Lateral and coronal

A

Answer: B
A trimalleolar fracture includes fracture of both malleoli and the posterior rim of the tibia.
Anteroposterior (AP) view of the ankle demonstrates the distal tibia and fibula, including the medial and lateral malleoli and the head of the talus. The fractures of both malleoli will be visible with this view. The lateral view provides evidence of the fracture at the posterior rim of the distal tibia.

177
Q
  1. A patient presents with pain and muscle spasm of the upper back (C7-T8) extending to the lateral border of the scapula. This encompasses a 10 × 10-cm area on both sides of the spine. If the US unit only has a 5-em? sound head, how should the therapist treat the upper back area?
    A. Esch side, allotting 5 minutes for each section
    B. The entire area in 5 minutes
    C. The entire area in 10 minutes
    D. Each side, allotting 2.5 minutes for each section
A

Answer: A
The total treatment area is too large for the 5-cm? sound head to produce adequate tissue heating. Sonating the two areas independently will allow more time for the tissue temperature to rise during the treatment time in each area.

178
Q

The physical therapist is examining the muscle length of the patient’s right hip. What is the muscle length test pictured below?
A. Ober test
B. FABER test
C. Thomas test
D. Noble test

A

Answer: A
The Ober test is for tightness of the tensor fascia late and iliotibial band. With the pelvis stabilized and the hip kept in neutral rotation, the Ober test would be considered normal if the thigh drops slightly below horizontal.

179
Q

A therapist is examining a patient’s balance using posturography testing with the Clinical Test for Sensory Integration in Balance (CTSIB). The patient’s sway increases with loss of balance under conditions with the eyes closed and platform moving (condition 5). During condition 6, with both the visual surround and platform moving, loss of balance is more immediate. Which of the following BEST identifies the source of the patient’s problems?
A. Vestibular deficiency
B. Visual dependency
C. Problems with sensory selection
D. Somatosensory dependency

A

Answer: A
The CTSIB is positive for vestibular deficiency with loss of balance on conditions 5 and 6.

180
Q

A patient presents with severe, frequent seizures originating in the medial temporal lobes.
After bilateral surgical removal of these areas, the patient is unable to remember any new information from just prior to the surgery to the present. The patient cannot recall test read minutes ago or remember people previously met. How should this surgical outcome be characterized?
A. Loss of integration of the temporal lobe with the basal ganglia and frontal cortex
B. Loss of procedural memory and integration with frontal cortex
C. Loss of the hippocampus and declarative memory function
D. A primary deficit from the loss of the amygdala

A

Answer: C
Declarative memory refers to conscious, explicit, or cognitive memory. It is a function of the cerebral cortex and the hippocampus.

181
Q

A 14-year-old with a body mass index of 33 kg/m? and a history of limited participation in physical activities is referred for exercise training. The nutritionist has prescribed a diet limiting his caloric intake. What Is the BEST initial exercise prescription for this patient?
A. Three weekly sessions of 60 minutes at 50% VO2max
B. Three weekly sessions of 50 minutes at 75%-85% VO2max
C. Three weekly sessions of 30 minutes at 65%-70% VO2max
D. Two daily sessions of 30 minutes at 45%-70% VO2max

A

Answer: D
This individual is obese (body mass index ≥ kg/m?) and will benefit from exercise to increase energy expenditure and diet to reduce caloric intake. The initial exercise prescription should utilize low-intensity with longer duration exercise. Splitting the training into two sessions each day is a good choice. The goal is to work toward bringing the target HR into a suitable range.
Obese individuals are at increased risk of orthopedic injuries and require close monitoring.

182
Q

A patient is admitted to a hospital after a fall. A review of the patient’ medical chart reveals a blood pressure (BP) of 160/85, a triglyceride level of 160 mg/dL, and a fasting blood glucose level of 115 mg/dL. Weight is 310 Ib. examination of the patient reveals a rotund man with a 54-inch waistline. Which diagnosis is consistent with this patient’s signs and symptoms?
A. Type 1 dibetes
B. Cushing’s syndrome
C. Metabolic syndrome
D. Chronic heart disease

A

Answer: C
This patient is exhibiting four of the risk factors of metabolic syndrome (diagnosis is made if three or more are present). Risk factors include (1) abdominal obesity: waist circumference > 40
inches in men or > 35 inches in women; (2) elevated triglycerides: triglyceride level of 150 mg/dL or higher; (3) low high density lipoprotein (HDL) cholesterol or being on medicine to treat low HDL: HDL level < 40 mg/dL in men or 50 mg/dL in women; (4) elevated BP: systolic BP ≥ 130 mm Hg and/or diastolic BP = 85 mm Hg; and (5) fasting plasma glucose level > 100 mg/dL. The therapist’s plan of care should be reflective of the patient’s increased risk for heart disease, stroke, and diabetes and should assist the patient in lifestyle changes that reduce these risk factors.

183
Q

A patient in an exercise class develops muscle weakness and fatigue. Examination reveals leg cramps and hyporeflexia. The patient also experiences frequent episodes of postural hypotension and dizziness. Abnormalities on the ECG include a flat T wave, prolonged QT interval, and depressed ST segment. Which electrolyte imbalance is consistent with this patient’s signs and symptoms?
A. Hypocalcemia
B. Hyperkalemia
C. Hypokalemia
D. Hyponatremia

A

Answer: C
Hypokalemia, decreased potassium in the blood, is characterized by these signs and symptoms.
Other possible symptoms include respiratory distress, irritability, confusion or depression, and gastrointestinal disturbances.

184
Q

A patient has experienced swelling in both lower legs (below the knees) since the age of
16 (10-year history). The referring diagnosis is bilateral lymphedema. Which test should the therapist include in the initial examination>
A. Ankle-brachial index (ABI)
B. Systolic blood pressure before and after exercise
C. Homan’s sign
D. Rubor of Dependency test

A

Answer: A
The ABI is helpful to determine the arterial patency or sufficiency. A value of 1.0 indicates normal arterial flow; values between 0.5 and 0.8 indicate moderate compromise while values below 0.5 indicate severe compromise of arterial flow. Adequate arterial blood flow is essential in considering application of compression for lymphedema.

185
Q

A patient is referred to a woman’s health specialist PT with a diagnosis of pelvic and uterine prolapse. Which of the following interventions is the BEST choice for this patient?
A. External stabilization with a support belt
B. Gentle abdominal exercises with incisional support
C. Protective splinting of abdominal musculature
D. Kegel’s exercises

A

Answer: D
Intervention should focus on pelvic floor rehabilitation (Keel’s exercises to strengthen pubococcygeal muscles) along with postural education and muscle re-education.

186
Q

A therapist is working with a patient recovering from traumatic brain injury, Rancho Los Amigos Levels of Cognitive Functioning (LOCF), level IV. This is the patient’s first time in the physical therapy gym. While the therapist attempts to work with the patient on sitting control, the patient becomes agitated and combative. Which strategy is the BEST choice in the situation?
A. Remove the patient to a quiet environment and provide support and calming stimuli
B. Give the patient a 5-minutes test and resume the training activity
C. Return the patient to his/her room and try the training activity later in the day
D. Enlist the help of a PTA to support the patient while slowly instructing the patient again

A

Answer: A
The best choice is to remove the patient to a quiet room and provide calming stimuli and support. At level IV LOCF, the therapist can expect the patient to be confused, inappropriate, and highly distractable. The environment should be changed. Memory is severely impaired and the patient is unable to learn new information.

187
Q

A patient has a 10-year history of Parkinson’s disease and has been on levodopa (carbidopa) for the past 6 years. The patient has fallen three times in the past month, resulting in a Colles” fracture. The therapist decides to try postural biofeedback training using a platform balance training device. Which of the following is the BEST choice for a training protocol?
A. Increase the limits of stability and improve anterior weight displacement
B. Decrease the limits of stability and anterior weight displacement
C. Increase the limits of stability and improve center of pressure alignment
D. Decrease the limits of stability and improve posterior weight displacement

A

Answer: C
The patient with Parkinson’s disease exhibits significant balance impairments including loss of postural reflexes; decreased limits of stability; flexed, stooped posture that alters the center of pressure in an anterior direction; freezing; and orthostatic hypotension. Platform balance training should work toward improving the limits of stability and center of pressure alignment (the patient should focus on reducing anterior displacement).

188
Q
  1. A patient is recovering from MI and is referred for supervised exercise training. While working out on a treadmill, the patient begins to develop mild shortness of breath.
    Continuous telemetry monitoring reveals the following ECG rhythm. What is the correct interpretation of this rhythm?
    A. Tachycardia with abnormal P waves
    B. Sinus rhythm with upsloping ST segment depression
    C. Sinus rhythm with sinoatrial blocks
    D. Sinus rhythm with downsloping ST segment depression
A

Answer: B
This patient is exhibiting sinus rhythm with upsloping ST segment depression. An abnormal ECG response is defined as ≥ 1 mm of horizontal or downsloping at 80 sec beyond the J point. Associated clinical signs suggestive of myocardial ischemia include dyspnea and angina.
After a review of this patient’s exercise responses coupled with the ECG findings, the therapist correctly determines that the exercise session does not have to be terminated. The therapist should continue to closely monitor the patient’s responses.

189
Q

A female patient complains of right lumbosacral pain after giving natural childbirth to her first child 2 months ago. Pain has subsided somewhat, but remains high enough that she has to sit after walking more than 2 blocks. Pain is noted in the right lumbosacral region buttock, and groin and is aggravated with weight bearing on the right. Active flexion, extension, and side bending reproduce the patient’s symptoms. Hamstrings are slightly tight on the right, but no neutral tension is noted. Neurological findings (reflexes, sensation, and motor) are unremarkable. SI provocation tests are positive. What is the MOST likely diagnosis for this patient?
A. Lumbar disc protrusion at LS/SI
B. Sacroiliac sprain
C. Quadratus lumborum strain
D. Piriformis syndrome

A

Answer: B
Child-bearing can place considerable stress on the sacroiliac joint, causing an overstretch of the ligaments. Pain is reproduced with active movement testing, which is characteristic of a musculoskeletal condition. Sacroiliac provocation tests are fairly valid as well, thus implicating the sacroiliac joint. The groin and buttock are common referral regions for the sacroiliac joint.

190
Q

A patient presents with a complaint of severe neck and shoulder pain of 2 days’ duration.
The patient reports falling asleep on the couch watching TV and has been stiff and sore since. There is tenderness of the cervical muscles on the right, with increased pain upon palpation. Passive ROM is most limited in flexion, then side-bending left, and then rotation left and active extension. Side-bending right and rotation right are also painful. What is the MOST likely diagnosis for this patient?
A. Cervical strain
B. Herniated disc
C. Cervical radiculopathy
D. Facet syndrome

A

Answer: D
A facet syndrome presents with localized pain.

191
Q

A postal worker (mail sorter) complains of numbness and tingling in the right hand.
Examination reveals a median nerve distribution. When the therapist evaluates the patient’s work tasks, the therapist notes that the patient is required to key in the zip codes of about 58 letters per minute. Which of the following is the BEST choice of administrative control to reduce this patient’s problem?
A. Require the worker to attend a cumulative trauma disorder educational class
B. Provide the worker with a resting splint to support the wrist
C. Provide a height adjustable chair to position the wrists and hands in a neutral alignment
D. Use job rotation during the workday

A

Answer: D
Administrative controls reduce the duration, frequency, and severity of exposures to ergonomic stressors. Job rotaion reduces fatigue and stress by rotating the worker to jobs that use different muscle-tendon groups during the workday.

192
Q

A patient with a traumatic brain injury presents with hemiparesis. The examination reveals slight cutaneous and proprioceptive impairment, fair (3/5) strength of the shoulder muscles and triceps, and slight spasticity of the biceps. Voluntary control of the patient’s left arm has not progressed since admission. The therapist decides to use functional electrical stimulation (FES), placing the active electrode on the triceps to facilitate active extension of the elbow. Which of the following is the BEST choice of timing sequence for FES in this case?
A. No ramp up, 10-second stimulation, 2-second ramp down
B. 2-second ramp up, 10-second stimulation, no ramp down
C. 2-second ramp up, 5-second stimulation, 2-second ramp down
D. 5-second ramp up, 5-second stimulation, 5-second ramp down

A

Answer: D
A relatively long ramp-up time over a 5-second period is used to minimize stimulating the muscle too quickly and increasing the spasticity. The ramp-down time has no effect on spasticity.

193
Q

After a traumatic brain injury, a patient presents with significant difficulties in learning how to use a wheelchair. Memory for new learning is present but limited (Rancho Los Amigos Levels of Cognitive Functioning, level VII). The patient is wheelchair dependent and needs to learn how to transfer from the wheelchair to the mat (a skill never done before).
Which of the following is the BEST strategy to enhance this patient’s motor learning?
A. Use only guided movement to ensure correct performance
B. Provide bandwidth feedback using a random practice schedule
C. Provide consistent feedback using a blocked practice schedule
D. Provide summed feedback after every few trials using a serial practice schedule

A

Answer: C
Early learning should focus on consistent feedback given after every trial to improve initial performance. A blocked practice schedule with repeated practice of the same skill will also reinforce early learning.

194
Q

A patient recovering from stroke presents with predominant involvement of the contralateral lower extremity and lesser involvement of the contralateral upper extremity.
These clinical manifestations are characteristic of which cerebral syndrome?
A. Basilar artery syndrome
B. Anterior cerebral artery syndrome
C. Posterior cerebral artery syndrome
D. Middle cerebral artery syndrome

A

Answer: B
These clinical manifestations are consistent with anterior cerebral artery syndrome (lower extremity is more involved that upper extremity).

195
Q

A patient presents with significant intermittent claudication with onset after 2 minutes of walking. Which of the following are likely findings for this patient, given the patient’s symptoms in response to walking?
A. Persistent local redness of the extremity in both gravity-dependent and independent positions
B. Elevation-induced pallor and dependent redness with the extremity in the gravity-dependent position
C. Little or no changes in color with changes in extremity position
D. A brownish color just above the ankle in both gravity-dependent and independent positons.

A

Answer: B
Intermittent claudication (episodic muscular ischemia induced by exercise) is due to obstruction of large or middle-sized arteries by atherosclerosis. The Rubor of Dependency test is used to assess the adequacy of arterial circulation by evaluating the skin color changes that occur with first extremity elevation (pallor) and then lowering of the extremities (delayed color changes, redness.

196
Q

After a recent caesarean, a patient tells the therapist that she is anxious to return to her prepregnancy level of physical activity (working out at the gym three days/wk and running 5 miles every other day). What is the BEST choice of activities for this patient at this time?
A. Pelvic floor and gentle abdominal exercises for the first 4-6 weeks
B. A walking program progressing to running after 4 weeks
C. Pelvic floor exercises and refrain from all other exercise and running for at least 12
weeks
D. Abdominal crunches with return to running after 5 weeks

A

Answer: A
Postcesarean physical therapy can include postoperative TENS, assisted breathing and coughing techniques, and gentle abdominal exercises with incisional support provided by a pillow. Pelvic floor exercise are also important because hours of labor and pushing are typically present before surgery.

197
Q

A 26-year-old who was diagnosed with schizophrenia, disorganized type, at the age of 22 is referred for gait training after a compound fracture of the tibia. The therapist suspects the patient has recently experienced an exacerbation of schizophrenia. Which behaviors support this conclusion?
A. Poor ability to perform multistep tasks requiring abstract problem-solving
B. Sleep disturbances and flashbacks
C. Increased fear of going out in public
D. Frequent verbalizations of pervasive feelings of low-self-esteem

A

Answer: A
Schizophrenia is characterized by disordered thinking (fragmented thoughts, errors of logic or abstract reasoning, delusions, poor judgment, and so forth).

198
Q

The rehabilitation team is completing a home visit to recommend environmental modifications for a patient who is scheduled to be discharged next week. The patient is wheelchair dependent. The home has not been adapted. Which of the following recommendations is correct?
A. Widening the door entrance to 28 inches
B. Adding horizontal grab bars in the bathroom positioned at 34 inches
C. Raising the toilet seat to 25 inches
D. Installing an entry-way ramp with a running slope of 1:10

A

Answer: B
Horizontal grab bars should be positioned at an optimal height of 33-36 inches.

199
Q

During an examination of gait, the therapist observes lateral pelvic tilt on the side of the swing leg during frontal plane analysis. What is the purpose of the lateral pelvic tilt on the side of the swing leg during gait?
A. Reduce physiological valgum at the knee
B. Reduce knee flexion at mid-stance
C. Control forward and backward rotations of the pelvis
D. Reduce peak rise of the pelvis

A

Answer: D
Lateral pelvic tilt in the frontal plane keeps the peak of the sinusoidal curve lower than it would have been if the pelvis did not drop. Lateral pelvic tilt to the right is controlled by the left hip abductors.

200
Q

A group of researchers investigated the effect of tai chi on perceived health status in older, frail adults. The subjects were 269 women who were older than 70 years of age and recruited from five independent senior living facilities. Participants took part in a 48-week single-blind RCT. Perceived health status was measured by five pretrained testers using the Sickness Impact Profile (SIP) the researchers found significant perceived health benefits.
Which of the following is an accurate conclusion regarding the design of this study?
A. Generalizability to a larger population of elderly women
B. Errors in reliability due to the number of testers
C. Errors in validity due to the selection of the outcome measure
D. Limited findings on the effects of tai chi exercise in the frail elderly

A

Answer: A
This study provides important findings on the effects of tai chi in elderly women. It is an RCT with a large number of subjects from multiple centers. Thus it has good generalizability to a population of elderly women. The SIP is a gold standard instrument with extensive testing and established validity and reliability.